Recalls 5

You might also like

Download as pdf or txt
Download as pdf or txt
You are on page 1of 27

6. With your knowledge on vaccine shelf-life and a. Red, Yellow, Green c.

Cyclical
RECALLS 5 – NP1 storage, which should NOT be stored in the freezer?
a. DPT
b. Pink, Yellow, Green
c. Red, Orange, Green
d. Secular variation
Situation
b. Oral polio vaccine d. Red, Pink, Green
You are a newly hired community nurse in Barangay 17. Around 50% of the affected individuals reported
c. Measles vaccine
Guada. You were tasked to provide vaccination to the
d. MMR 12. In Integrated Management of Childhood Illness, vomiting more than once in the past 3 hours. Which is
citizens.
severe conditions generally require urgent referral to a your priority nursing problem for these individuals?
1. You have a role in improving the delivery of
7. When handling, transporting and storing vaccines, hospital. Which of the following severe conditions does a. Risk for electrolyte imbalance
immunization services in the community. As a nurse,
special care must be given to provide quality potent not always require urgent referral to a hospital? b. Imbalanced nutrition: Less than body requirements
you need to:
vaccines among the targets. What measure is practiced a. Mastoiditis c. Activity intolerance
a. Record the children given with vaccination in the
to ensure that all vaccines are utilized before its b. Severe dehydration d. Impaired comfort
Target Client list and GECD/GMC card or any standard
expiration? c. Severe pneumonia
recording form utilized.
a. FIFO d. Severe febrile disease 18. You gathered data on how many people were
b. Identify cases of EPI target diseases per standard case
b. FEFO affected and presented the trend with the time people
definition
c. Most sensitive to heat to Least sensitive to health 13. A 5-month old infant was brought by his mother complained to the health center regarding their
c. Conduct health visits in the community to assess
d. Potency arrangement to the health center because of diarrhea occurring 4 to symptoms. What type of graph will you use?
other health needs of the community and be able to
5 times a day. His skin goes back slowly after a skin a. Bar chart
provide package of health services to targets
8. A mother asks you, “Why is BCG needed for my pinch and his eyes are sunken. Using the IMCI b. Histogram
d. All of the above
newborn?” Your answer would be: guidelines, you will classify this infant in which c. Pie chart
a. “BCG protects the possibility of TB meningitis & other category? d. Line graph
2. The Philippine Expanded Program on Immunization
TB infections.” a. No signs of dehydration
(EPI) has been in existence since 1976, providing Filipino
b. “BCG reduces the chance of severe pertussis.” b. Some dehydration 19. Which of the following traditional medicines may
children access to safe and effective vaccines to protect
c. “BCG prevents liver cirrhosis and liver cancer.” c. Severe dehydration be used for diarrhea and stomach pains?
them from diseases like measles, diphtheria, tetanus,
d. None of the above. d. The data is insufficient. a. Sambong
and whooping cough. Which Republic Act is this?
a. RA 9211 b. Yerba buena
9. What day of the week is designated as 14. Management of a child with measles includes the c. Tsaang gubat
b. RA 9165
immunization day in the Philippines? administration of which of the following? d. Lagundi
c. RA 10152
a. Monday a. Gentian violet on mouth lesions
d. RA 10586
b. Tuesday b. Antibiotics to prevent pneumonia 20. You aim to provide health education to the people
c. Wednesday c. Tetracycline eye ointment for corneal opacity regarding proper food preparation techniques to
3. You will not give DPT 2 if the mother says that the
d. Friday d. Retinol capsule regardless of when the last dose was prevent food-borne illnesses. Which of the following
infant had:
given recommendations will you not include in your health
a. Seizures a day after DPT 1.
10. Baby Julian is scheduled to have his third doses of teaching plan?
b. Fever for 3 days after DPT 1.
OPV, PCV, and Pentavalent vaccine and first dose of IPV 15. A 6-month old infant was brought to the health i. Wash hands before handling food.
c. Abscess formation after DPT 1.
on August 2, 2023. Today is July 15, 2023. However, his center because of cough. Her respiratory rate is ii. Store raw and prepared food together.
d. Local tenderness for 3 days after DPT 1.
mother informed you that they will go to Korea on July 45/minute. Using the Integrated Management of Child iii. Do not leave cooked food at room
30 to August 5, 2023. What will you prescribe for Baby Illness (IMCI) guidelines of assessment, her breathing is temperature for more than 3 hours.
4. A mother calls the pediatric clinic to ask when her
Julian’s immunization? considered: iv. Reheat cooked food thoroughly.
daughter will receive the Varicella vaccine. Your answer
a. Give the vaccine today a. Fast v. Thaw frozen food at room temperature.
to her question is:
b. Go back to the health center on July 28, 2023 b. Slow a. i, iv
a. at 2, 4, and 6 months
c. Go back to the health center on August 8, 2023. c. Normal b. ii, iii
b. 12 months and 4-6 years
d. Cancel their Korea trip d. Insignificant c. ii, iii, v
c. at 6 and 12 months
d. at 4 months and 4-6 years d. None of the above
Situation Situation
The IMCI or Integrated Management of Childhood Barangay Casita had a health and medical caravan for Situation
5. A pregnant woman had just received her 4th dose
Illness (IMCI) is a strategy that integrates all available elderly patients. While waiting, they were given a ham You want to be a community health nurse. The
of tetanus toxoid. Subsequently, her baby will have
measures for disease prevention and health problems and cheese sandwich and a bottle of orange juice. The following questions apply in your desired practice.
protection against tetanus for how long?
during childhood, for their early detection and effective caravan ended by lunch time. However, at 3PM, several
a. 1 year
treatment, and for promoting healthy habits within the people went back to the health center and complained 21. Population-focused nursing practice requires
b. 3 years
family and community. The following questions apply. of stomach pain, vomiting, and diarrhea. which of the following processes?
c. 10 years
d. Lifetime 16. What type of epidemic is the situation above? a. Community organizing
11. What colors are being utilized in the color coded a. Point source b. Nursing process
system of the IMCI? b. Propagated c. Community diagnosis
d. Epidemiologic process c. Outcome 33. The nurse is teaching a patient taking rifampicin. you can give to women in the first trimester of
d. Impact Which side effect is expected with this drug?
22. Which of the following is the mission of the a. Hypertension pregnancy in the barangay?
Department of Health? 28. What element is being evaluated is the nurse b. Reddish brown urine a. Advice them on the signs of German measles.
a. Health for all Filipinos assessing the budget rendered and equipment used in c. Change in hearing abilities b. Avoid crowded places, such as markets and cinemas.
b. Ensure the accessibility and quality of health care the nursing services given? d. Joint pain c. Consult at the health center where a rubella vaccine
c. Improve the general health status of the population a. Process may be given.
d. Health in the hands of the Filipino people by the year b. Structural 34. A patient asks you, “Why do I need to take d. Consult a physician who may give them rubella
2020 c. Outcome multiple drugs at the same time?” Your most immunoglobulin.
d. Impact appropriate answer is:
23. Which of the following is the most prominent a. “Multiple drugs help so that you will not need high 39. What is your best advice for the community to
feature of public health nursing? 29. What element is being evaluated is the nurse doses of each drug.” prevent transmission of measles?
a. It involves providing home care to sick people who assessing the nursing care plan formulated and how it is b. “Multiple drugs help prevent drug resistance.” a. Always observe sanitary food preparation techniques.
are not confined in the hospital. implemented? c. “Multiple drugs help in avoiding adverse effects.” b. Cover your mouth with tissue paper when coughing.
b. Services are provided free of charge to people within a. Process d. “Multiple drugs help to maximize the antibacterial c. Avoid sharing utensils with affected individuals.
the catchment area. b. Structural actions of each.” d. Get MMR vaccine.
c. The public health nurse functions as part of a team c. Outcome
providing public health nursing services. d. Impact 35. A pregnant mother came to you with a persistent 40. Paragonimiasis is a chronic parasitic infection. The
d. Public health nursing focuses on preventive, not cough and fatigue. After necessary tests, she was following are correct about paragonimiasis, except:
curative, services. 30. When evaluating the RHU staff in terms of their diagnosed with TB and the causative agent is a. Paragonimiasis present with PTB symptoms.
performance of job responsibilities, what tool should mycobacterium tuberculosis. Which instruction is b. Paragonimus westermani is the most important
you utilize? included in your teaching plan? causative agent in Asia.
24. Who is the Chairman of the Municipal Health a. Performance evaluation a. Medication will be started once the baby is delivered. c. Paragonimiasis is diagnosed through stool specimen
Board? b. Incident report b. Pregnancy has to be aborted. examination.
a. Mayor c. Nursing audit c. She will take rifampicin, pyrazinamide, and d. Paragonimiasis is transmitted through ingestion of
b. Municipal Health Officer d. Staff meeting streptomycin for 9 months. raw crabs.
c. Public Health Nurse d. She will take rifampicin and isoniazid for 9 months.
d. Any qualified physician Situation 41. What would you expect in patients with
Barangay Gapan has a high incidence of Tuberculosis. Situation Paragonimiasis?
25. It is a means of mobilizing people to solve their You were asked by the RHU to provide services to help There is an outbreak of communicable diseases a. Hemoptysis
own problems. manage, prevent and control the disease in their reported in different provinces across the country. The b. Fever
a. Community diagnosis community. following questions apply. c. Rashes
b. Community organizing d. Jaundice
c. Community assessment 31. Which clients are considered targets for DOTS 36. A 4-year old client was brought to the health
d. Community participation Category I? center with the chief complaint of severe diarrhea and 42. What is the drug of choice for Paragonimiasis?
a. Sputum negative cavitary cases the passage of “rice water” stools. The client is most a. Bithionol
26. Which of the following actions show the b. Clients returning after a default probably suffering from which condition? b. Praziquantel
supervisory function of a public health nurse? c. Relapses and failures of previous PTB treatment a. Giardiasis c. Erythromycin
a. Providing home visits to chronically ill patients in the regimens b. Cholera d. Chloroquine
community d. Clients diagnosed for the first time through a positive c. Amebiasis
b. Coordinating care with the members of the RHU sputum exam d. Dysentery 43. Cholera is caused by a bacterium called Vibrio
c. Lobbying advocacy for healthier environment and cholerae. The symptoms are produced due to a toxin
lifestyle for the citizens 32. To improve compliance to treatment, what 37. Several children have measles. The released by the bacteria.
d. Coaching BHWs how to conduct vital signs innovation is being implemented in DOTS? pathognomonic sign of measles is Koplik’s spot. You a. Bacteria
assessment a. Having the health worker follow up the client at may see Koplik’s spot by inspecting the _____. b. Virus
home a. Nasal mucosa c. Protozoa
27. To assess the effectiveness of provided care and b. Having the health worker or a responsible family b. Buccal mucosa d. Fungus
services, a public health nurse conducts evaluation in member monitor drug intake c. Skin on the abdomen
every nursing activity. What element is being evaluated c. Having the patient come to the health center every d. Skin on the antecubital surface 44. What is the priority nursing intervention should
is the nurse assessing the changes in the client's health month to get his medications
you provide to a patient with cholera?
status that result from nursing intervention? d. Having a target list to check on whether the patient
38. A barangay had an outbreak of German measles. To a. Give ORESOL
a. Process has collected his monthly supply of drugs
b. Weigh the patient daily
b. Structural prevent congenital rubella, what is the BEST advice that c. Obtain stool specimen
d. Ensure adequate food intake prior to consultation. To determine malaria risk, what b. Inability to make decisions with respect to taking 60. Liah asks you what medicine she should take to
will you do first? appropriate health action due to low salience of the manage her PCOS. What would be you most
45. In the Philippines, which condition is the most a. Do a tourniquet test. problem appropriate response?
frequent cause of death associated with b. Ask where the family resides. c. Inability to provide a home environment conducive to a. You are not recommended to take any pills because
schistosomiasis? c. Get a specimen for blood smear. health maintenance and personal development due to of your hormonal imbalances.
a. Bladder cancer d. Ask if the fever is present everyday. lack of knowledge of preventive measures b. You may take any brand of progesterone-only pills
b. Liver cancer d. Inability to provide adequate nursing care to the at (POPs).
c. Liver cirrhosis Situation risk member of the family due to lack of knowledge in c. The health center offers contraceptive pills. You can
d. Intestinal perforation You are caring for Family Cruz, which consists of Rey carrying out necessary interventions ask for their available pills for free.
(father, 46 y/o), Joy (mother, 43 y/o), and 4 kids with d. Let us talk to your ob-gyne regarding the medication
46. Mosquito-borne diseases are prevented mostly ages 21, 16, 14, 8. The following questions apply. 55. What topic should you teach first to Rey? you should take.
with the use of mosquito control measures. Which of a. How smoking can lead to serious diseases
the following is NOT appropriate for malaria control? 51. You plan to visit the family in their home. Which b. Techniques to lessen cravings for smoking Situation
a. Use of chemically treated mosquito nets of the following is an advantage of a home visit? c. Resources for smoking cessation Environmental health is also an important aspect when
b. Seeding of breeding places with larva-eating fish a. It allows the nurse to provide nursing care to a d. Withdrawal symptoms to expect if he quit smoking caring for a community. As a nurse, you are tasked to
c. Destruction of breeding places of the mosquito vector greater number of people. address the environmental issues related to health in
d. Use of mosquito-repelling soaps, such as those with b. It provides an opportunity to do first hand appraisal 56. Which of the following objectives is most Brgy UNO.
basil or citronella of the home situation. appropriate for Rey?
c. It allows sharing of experiences among people with a. Rey will stop smoking starting next week. 61. When caring for the environment in relation to
47. An important role of the community health nurse similar health problems. b. Rey will only smoke ½ pack in the next 4 weeks. health, a nurse follows the Man-Disease Agent-
in the prevention and control of Dengue H-fever d. It develops the family’s initiative in providing for the c. Rey will gradually decrease his smoking until he can Environment Triad. All but one are preventive strategies
includes: health needs of its members. completely stop smoking that apply this triad:
a. Advising the elimination of vectors by keeping water d. Rey will decrease his smoking by 2 sticks every 3 days a. Measles immunization
containers covered 52. The typology of family nursing problems is used in for the next two months. b. Treatment of wastewater
b. Conducting strong health education drives/campaign the statement of nursing diagnosis in the care of c. Food safety practices
directed toward proper garbage disposal families. Rey is a smoker for 15 years and drinks with his 57. Furthermore, their 16-year old son is about to d. None of the above
c. Explaining to the individuals, families, groups and compadres 4 nights a week. His favorite pulutan is start studying in a private university for Senior High
community the nature of the disease and its causation. chicharon. This is classified as a: School. The family belongs in the lower middle-income 62. If Brgy UNO is an endemic area for
d. Practicing residual spraying with insecticides a. Health deficit class and only has enough financial resources to sustain schistosomiasis, what is the most effective way of
b. Health threat their needs. This is classified as a: controlling schistosomiasis would you suggest?
48. Which of these signs may not be regarded as truly c. Foreseeable crisis a. Wellness condition a. Use of molluscicides
positive signs indicative of Dengue H-fever? d. Stress point b. Health deficit b. Building of foot bridges
a. Prolonged Bleeding Time c. Health threat c. Proper use of sanitary toilets
b. Appearance of at least 20 petechiae within 1 cm 53. You plan on creating a health teaching plan for d. Foreseeable crisis d. Use of protective footwear, such as rubber boots
square Rey. Upon assessment, you learned that he smokes 2
c. Steadily increasing hematocrit count packs per day for 5 years. Slowly, he limits himself to 1 58. Their 21-year old daughter, Liah, was diagnosed 63. Bgry UNO gets their water from a communal
d. Fall in the platelet count pack per day until today. How many pack years does with Polycystic Ovarian Syndrome faucet, where one faucet is used by 4-6 households.
Rey smoke? (PCOS) 3 years ago. You know that estrogen levels are What level of water supply facility is this?
49. A 3-year old child was brought by his mother to a. 10 elevated in PCOS. This increases her risk of what a. I
the health center because of a fever with a 4-day b. 15 disease? b. II
duration. The child had a positive tourniquet test result. c. 20 a. Metabolic syndrome c. III
In the absence of other signs, which is the most d. 25 b. Endometrial cancer d. IV
appropriate measure that the PHN may carry out to c. Hirsutism
prevent Dengue shock syndrome? 54. When you asked Rey regarding smoking cessation, d. Hypertension 64. Food fortification is one of the strategies to
a. Insert an NGT and give fluids per NGT. he answered, “Smoking is bad; I saw it in my prevent micronutrient deficiency conditions. R.A. 8976
b. Start the patient on IV fluid STAT. compadres. But I have no problems with it because as 59. You ask Liah regarding her lifestyle. Which of the mandates fortification of certain food items. Which of
c. Instruct the mother to give the child Oresol. you can see I am still healthy and strong.” What second following behaviors need to be addressed? the following is among these food items?
d. Refer the client to the physician for appropriate level assessment can you apply to this nursing problem? a. She walks every day at 6 AM for 30 minutes. a. Sugar
management. a. Inability to recognize the presence of the problem b. She sleeps at 11PM every night. b. Bread
due to denial about its existence as a result of fear of c. She eats 2 pieces of bread with jam for breakfast. c. Margarine
50. A mother brought her 10 month old infant for consequences of a diagnosis d. She occasionally drinks alcohol at least once a month. d. Filled milk
consultation because of fever, which started 4 days
65. Food sanitation is important to prevent food- a. “Tell me about it.” a. Health programs are sustained according to the level a. Crude birth rate
borne illnesses. When reheating food before eating, it b. “Yes, you should not have sex with him!” of development of the community. b. Neonatal mortality rate
should be up to at least what centigrade? c. “I can see that you are frustrated. Do you think the b. Health services are provided free of charge to c. Infant mortality rate
a. 50 intercourse caused your symptoms?” individuals and families. d. General fertility rate
b. 60 d. “I can’t believe he does that. Men like him do not c. Local officials are empowered as the major decision
c. 70 deserve to get near other people.” makers in matters of health. Situation
d. 80 d. Health workers are able to provide care based on A new public health nurse intends to improve the
70. She asks you to check for his records because she identified health needs of the people. lifestyle of the community by giving health education
Situation wants to know if her classmate transmitted the disease through seminars and group discussion in her
HIV and AIDS has been a serious problem in the to her. In terms of legalities, which of the following 75. Which of the following demonstrates intersectoral barangay.
country. To effectively address it, controlling it at should Nurse Jane do? linkages? 81. What level of prevention does the nurse plan to
community level through health education and early a. Accept Kathryn’s request. a. Two-way referral system conduct?
screening is important. b. Refuse Kathryn’s request. b. Team approach a. Primary
c. Refer Kathryn to the Informations Department of the c. Endorsement done by a midwife to another midwife b. Secondary
66. To determine possible sources of sexually hospital. d. Cooperation between the PHN and public school c. Tertiary
transmitted infections, which is the BEST method that d. Advise Kathryn to ask her doctor regarding the teacher d. A and B
may be undertaken by the public health nurse? records.
a. Contact tracing 76. You are planning for a program against 82. Which refers to a social action process through
b. Community survey Situation hypertension in a community with 1490 adults. There which people gain mastery over their lives and their
c. Mass screening tests Public health services are provided to people. The are 690 males and 800 females. What is the sex ratio? communities and community members assume greater
d. Interview of suspects Philippines currently implements the Universal Health a. There are 86 males per 100 females power, or expand their power from within, to create
Care Law. b. There are 46% males in 1490 adults desired changes?
67. Antiretroviral agents, such as AZT, are used in the c. There are 100 males per 86 females a. Participation
management of AIDS. Which of the following is not an 71. “Public health services are given free of charge.” d. There is equal male to female ratio b. Social relevance
action expected of these drugs? Is this statement true or false? c. Critical consciousness
a. They prolong the life of the client with AIDS. a. The statement is true; it is the responsibility of the 77. Estimate the number of pregnant women who will d. Empowerment
b. They reduce the risk of opportunistic infections government to provide basic services. be given tetanus toxoid during an immunization
c. They shorten the period of communicability of the b. The statement is false; people pay indirectly for outreach activity in a barangay with a population of 83. RA 7160 mandates devolution of basic services
disease. public health services. about 1,850. from the national government to local government
d. They are able to bring about a cure of the disease c. The statement may be true or false, depending on the a. 475 units. Which of the following is the major goal of
condition. specific service required. b. 550 devolution?
d. The statement may be true or false, depending on c. 600 a. To strengthen local government units
68. You are the PHN in the city health center. A client policies of the government concerned d. 650 b. To allow greater autonomy to local government units
underwent screening for AIDS using ELISA. His result c. To empower the people and promote their self-
was positive. What is the best course of action that you 72. Which is true of primary facilities? 78. Estimate the number of children who will be given reliance
may take? a. They are usually government-run. pentavalent vaccine during an immunization outreach d. To make basic services more accessible to the people
a. Get a thorough history of the client, focusing on the b. Their services are provided on an out-patient basis. activity in a barangay with a population of about
practice of high risk behaviors. c. They are training facilities for health professionals. 50,000. 84. What best signifies that learning is effective?
b. Ask the client to be accompanied by a significant d. A community hospital is an example of this level of a. 1350 a. Community members continue to participate in the
person before revealing the result. health facilities b. 1500 Zumba session every morning
c. Refer the client to the physician since he is the best c. 1800 b. Community members are able to describe the
person to reveal the result to the client. 73. What is the legal basis for Primary Health Care d. 2000 important of a healthy lifestyle
d. Refer the client for a supplementary test, such as approach in the Philippines? c. Community members verbalize that they will apply
Western blot, since the ELISA result may be false. a. Alma Ata Declaration on PHC 79. In a barangay with a total population of about their learning
b. RA 11223 18,000, there were 94 deaths. Among those who died, d. Community members put posters on how to eat
69. Kathryn is 19 years old, a first year college c. Presidential Decree 996 20 died because of diseases of the heart and 32 were healthily in multiple areas around their barangay
student. She recently confessed to Nurse Jane that she d. Letter of Instruction No. 949 aged 50 years or older. What is the crude death rate?
had sexual intercourse with her classmate. After 1 a. 4.2/1,000 85. When assessing the people’s risk for Non-
week, she experienced on and off flu-like symptoms. 74. Primary health care is a total approach to b. 5.2/1,000 Communicable Diseases such as hypertension and
She said, “I should have not had sex with him! I think I community development. Which of the following is an c. 6.3/1,000 diabetes, what protocol can the public health nurse
have HIV now because he goes to clubs all the time and indicator of success in the use of the primary health d. 7.3/1,000 use?
engages in sex with people he just met.” What is the care approach? a. PhilPEN
most therapeutic response for Kathryn? 80. Which of the following is a natality rate?
b. Integrated Management of Adolescent and Adult 92. Nurse Stephanie makes an ocular survey to the b. Create an office in the barangay health center that
Illness barangay. It is appropriate for her to: she can visit everyday
c. IMCI a. Introduce herself to the community members c. Go to the public areas in the community and converse
d. FOURmula One for Health b. Visit the barangay captain with the people
c. Establish the ground of her organization d. Conduct house-to-house surveys
Situation d. Assess the community and its members
The Field Health Services and Information System 100. What is the indication that Nurse Stephanie can
(FHSIS) is the recording and reporting system in public 93. What activity will Nurse Stephanie conduct first in now phase out of the community?
health care in the Philippines. the pre-entry phase? a. Plans are already in place and implemented
a. Immersion b. The community members already develops trust in
86. What is the fundamental block of FHSIS? b. Social preparation the public health nurse
a. Family treatment record c. Preliminary social investigation c. CHWs are able to assume staff level functions
b. Tally report d. Continuous appraisal d. The LGU have accepted to address the pressing needs
c. Output report of the community
d. Target/client list 94. When will be the best time to work with the
termination phase in a community immersion?
87. To monitor clients registered in long-term a. Pre-Interview Phase
regimens, such as the Multi-Drug Therapy, which b. Orientation Phase
component will be most useful? c. Working Phase
a. Tally report d. Termination Phase
b. Output report
c. Target/client list 95. What is the role of Nurse Stephanie in community
d. Individual health record research?
a. Facilitator
88. Who produces the output report? b. Leader
a. Public Health Office c. Formulator
b. Regional DOH Office d. Officer
c. Rural Health Unit
d. Barangay Health Center 96. Which of the following approaches should Nurse
Stephanie best apply when conducting community
89. When creating a tally report for infectious organization?
diseases, how often is the most ideal? a. Universal Health Care Law
a. Weekly b. Devolution of health services
b. Monthly c. Community Mobilization
c. Quarterly d. Primary Health Care
d. Annually
97. Which step in community organizing involves
90. To ensure a standardized facility database in training of potential leaders in the community?
FHSIS, what is the best approach? a. Integration
a. Conduct tally as often as possible b. Core group formation
b. Create summaries of cases recorded c. Community organization
c. Double check the accuracy of data d. Community study
d. Use the same forms across facilities
98. In which step are plans formulated for solving
Situation community problems?
Nurse Stephanie wants to conduct a community a. Mobilization
participation research in Brgy Viejo. b. Action
c. Core group formation
91. What is the primary goal of COPAR? d. Community organization
a. Community mobilization
b. Community health development 99. What is the best way for Nurse Stephanie to
c. Community organizing imbibe the community way of life in Brgy. Viejo?
d. Community assessment a. Visit the community every 2 days
10. You are caring for 5 month old Baby John. Which c. Fear of having no privacy
RECALLS 5 - NP2 a. Vitamin K aids in the absorption of calcium and
phosphorus for newborns to develop healthy bones and
developmental milestone, if not performed by the
infant, should be further investigated?
d. Fear of pain
Situation
teeth. a. Rolling over
A nurse should be knowledgeable and skilled in caring 15. According to Kohlberg’s Stages of Moral
b. Vitamin K is a water-soluble nutrient that helps in b. Crawling
for families with a newborn.
repairing red blood cells and boosting the immune c. Pincer grasp Development, what would be the reason for conformity
system. d. Sitting without support to the rules for a 9-year-old child?
1. A newborn’s mouth should be wiped first then the
c. Vitamin K is used to prevent brain damage in a. The child conforms to the rules because of the
nose. What is the most accurate rationale behind this?
newborns because it is essential for brain development, Situation rewards given for good behavior.
a. Newborns breathe through their mouths
neural myelination, and cognitive function. Psychosocial development is crucial throughout the b. The child conforms to the rules because it maintains
b. Newborns breathe through their nose
d. Vitamin K is needed in the production of certain entire lifespan. It emphasizes the social nature of social order.
c. Newborns may inhale outside air
coagulation factors in the intestinal tract to prevent and human beings, develops one’s personality, and c. The child conforms to the rules because it causes
d. Newborns may drink their amniotic fluid
treat bleeding diseases in newborns. influences social relationships. others to view them as being good.
2. When the Moro Reflex is stimulated in an infant, d. The child conforms to the rules because it provides a
Situation 11. The school-age child experiences many physical mutual benefit.
the infant will _____________the arms with the palms
As a nurse, you should have adequate grasp on changes throughout this developmental time period.
of the hands turned ___________ and then move the
developmental milestones in order to effectively assess Which of the following is not a change experienced by 16. Jean Piaget (1896–1980), a Swiss psychologist,
arms ___________ the body.
and care for pediatric patients. this age group? introduced concepts of cognitive development or the
a. flex, upward, away from
b. extend, upward, back to a. The child typically grows about 2.5 inches per year. way children learn and think. According to his theory, all
6. Baby Julie weighed 8 lbs. at birth. How many lbs. b. Most of the child’s 28 permanent teeth will be but one are expected in an infant:
c. flex, downward, back to
should she weigh at 6 months? erupted by the end of this developmental period. a. Infants play peek-a-boo because they realize the
d. extend, downward, away from
a. 24 lbs c. The child’s weight triples from the age of 6 to 12 person playing with them exists behind his or her
b. 10 lbs years. hands.
3. Most newborns experience physiologic jaundice 24
c. 16 lbs d. The child’s gross motor skills allow them to ride a b. Infants can recognize that their toy has broken parts.
hours after delivery, as a result of breakdown of fetal
d. 32 lbs bike without training wheels, swim, and skate. c. Infants learn they are a separate entity from objects.
red blood cells. However, when indirect bilirubin levels
become above normal, this may be potentially d. Infants can recognize that a parent remains the same
7. You noted that Baby Casey is using the pincer grasp 12. Which of the following is the most important person whether dressed in a robe and slippers or pants
dangerous because it can:
at 10-month-old. Which option below best describes achievement in a school age child? and a T-shirt.
a. Interfere with the chemical synthesis of brain cells,
this milestone? a. Being able to control their urges to urinate or
resulting in permanent cell damage called kernicterus
a. She is able to grasp large objects with the palm and defecate 17. According to Piaget's Theory of Cognitive
b. Lead to inadequate levels of red blood cells, resulting
forefingers. b. Being able to learn their sexual identity through Development a 11-year-old child would be in the
in anemia
b. She is able to grasp small objects with the index awareness of genital area _________________ stage:
c. Impair peripheral circulation, leading to decreased
finger and thumb. c. Being able to establish good relationships with people a. Preoperational
tissue perfusion and cyanosis
c. She is able to grasp small and large objects with the of the opposite sex b. Concrete Operational
d. Cause build-up of urea under the skin, resulting in
middle finger and thumb. d. Being able to learn a skill or hobby c. Sensorimotor
uraemic pruritus
d. She is able to grasp large objects with the palm and d. Formal Operational
thumb. 13. What would be your best advice for parents of a
4. Hazel, who just delivered her baby 17 hours ago,
8. Baby Casey’s mother asks you about car school age child? 18. Which of the following cognitive abilities is
asked you: “Why does my baby’s head look weird?”
safety: “What is the best position of a car seat for my a. Allow the child to complete a project and make them expected in an 11-year old child?
What is your most appropriate response?
baby?” Your response is: feel rewarded for the accomplishment a. Centering
a. “Your baby’s head does look weird. Let us ask your
a. Forward-facing and back seat b. Praise the child’s ability to make decisions rather b. Decentering
doctor why.”
b. Forward-facing and front seat than judging the correctness of their decisions c. Abstract thinking
b. “It looks asymmetrical. Was there anything that
c. Rear-facing and front seat c. Allow the child to create bonds and friendship with d. Reversibility
happened to your baby’s head?”
d. Rear-facing and back seat peers of the opposite sex
c. “Your baby’s head was molded during labor and
delivery. This will go back to normal after a few days.” d. Provide visual stimulation for active child 19. Suppose Carl, 5 years old, tells you that his arm is
9. You then note that Baby Casey’s anterior fontanelle involvement asking for a warm blanket to get better. What type of
d. “I don’t know what you are saying. It does not look
is open. How would you document this finding? cognition is he using?
weird.”
a. Abnormal; should be closed at 2 months 14. A 10-year-old is being prepped for heart surgery a. Deductive reasoning
b. Abnormal; should be closed at 6 months to replace a heart valve. What fears or stressors does b. Magical thinking
5. Baby Nicole was delivered 1 hour ago. You are
c. Normal, should be closed at 12 months the nurse anticipate this child could experience during c. Inductive reasoning
tasked to give BCG vaccine, Hepatitis B vaccine, and
d. Normal; should be closed at 18 months this hospitalization? d. Sensorial thought
Vitamin K injection. What is the best rationale in giving
Vitamin K in newborns? a. Fear of strangers
b. Fear of punishment
20. Carl’s mother told you that she is concerned c. Chrisa inhales the Salmeterol first and then waits 1 arising from an abnormal shunting of blood throughout c. Administer lorazepam
because she saw her son masturbating while taking a minute before inhaling the Fluticasone. the heart. As the nurse, you know that a d. Call for help
bath. As a nurse, what would be your most appropriate d. Chrisa inhales the Fluticasone and immediately _____________ shunt is occurring in the heart due to
response? inhales the Salmeterol. the defect: 35. A patient with a history of epilepsy is taking
a. This is normal behavior for Carl. a. Right-to-left Phenytoin. The patient’s morning labs are back, and the
b. This behavior should begin by 10 years old. 25. All but one are expected signs and symptoms in b. Right patient’s Phenytoin level is 7 mcg/mL. Based on this
c. It is recommended that you have him checked with a patients with gluten sensitivity: c. Left finding, the nurse will?
psychiatrist. a. Cachexia d. Left-to-right a. Assess the patient for a rash
d. Carl may have a disease in his genitalia. b. Steatorrhea b. Initiate seizure precautions
c. Abdominal distention 30. Cess’s mother comes to you and says, “I am c. Hold the next dose of Phenytoin
Situation d. None of the above feeling scared for my daughter. Why does it need to d. Continue to monitor the patient
Chrisa is an 8-year-old child born with asthma and celiac happen to her? I don’t want to lose her.” What is your
disease. She was brought to the hospital with vital signs Situation most therapeutic response? Situation
of BP 100/70, PR 99, RR 30, and T 36.8 deg C. She has One of the most common congenital diseases are a. Stay silent. Education on sexuality and reproductive health is
wheezes, pallor, and rashes on her chest . cardiovascular diseases. Nursing interventions for b. “Stop worrying. It will not do you or your daughter important for individuals to have adequate
pediatric patients with cardiovascular diseases are any good.” understanding about their bodies and relationships,
21. You asked Chrisa’s mother about her allergies. crucial to ensure optimal health and recovery. c. “Why are you feeling that way?” decrease their risk-taking, and decrease the frequency
She responded that Chrisa is allergic to seafood, which d. “I can see that you are losing hope and you think your of unprotected sex.
the child accidentally ate prior to her asthma attack. 26. Dan, a 2 month old infant, was born with daughter will die.”
What drug should be immediately given to the patient tetralogy of fallot. Which statement below is incorrect 36. The Reproductive Health Law of the Philippines
for her allergic reaction? concerning how the blood normally flows through the Situation provides universal and free access to modern
a. Epinephrine heart? 5-month-old James was brought by his father to the contraceptive methods, mandates age- and
b. Albuterol a. Unoxygenated blood enters through the superior and hospital for insertion of an external ventricular drain to development-appropriate reproductive health
c. Montelukast inferior vena cava and travels to the left atrium. manage his hydrocephalus. The following situations education in government schools, and recognizes a
d. Theophylline b. The pulmonic valve receives blood from the right apply. woman’s right to postabortion care in the Philippines as
ventricle and allows blood to flow to the lungs via the part of the right to reproductive healthcare. Which of
22. What are the three processes behind asthma? pulmonary artery. 31. Which of the following signs and symptoms can the following pertains to the RH Law?
a. Increased bronchial secretions, alveolar collapse, c. The left atrium allows blood to flow down through you not expect in James? a. RA 8423
airway constriction the bicuspid valve (mitral) into the left ventricle. a. Delayed closure of fontanelles b. RA 10354
b. Bronchospasm, increased mucus production, upper d. Oxygenated blood leaves the left ventricle and flows b. Projectile vomiting c. RA 7305
airway obstruction up through the aortic valve and aorta to be pumped to c. High pitched cry d. RA 9262
c. Bronchospasm, inflammation of bronchial mucosa, the rest of the body. d. None of the above
and increased bronchial secretions
d. Abnormal production of mucus, blockage of exocrine 27. While feeding Dan, you notice the infant’s skin 32. James’s father asks you where external 37. 17 year-old Jen came to you to ask about
glands, airway constriction begins to have a bluish tint and the breathing rate has ventricular drains are inserted. What is the correct contraceptives. She has a history of vascular disease.
increased. Your immediate nursing action is to? answer? Which of the following options should you not
23. Which of the following breakfast meals should a. Continue feeding the infant and place the infant on a. Subarachnoid space
you give to Chrisa? oxygen. b. Lateral Ventricle recommend to her?
a. Oatmeal with banana and peanut butter b. Stop feeding the infant and provide suction. c. Epidural space a. Condoms
b. Whole wheat toast with fried egg and cheese c. Stop feeding the infant and place the infant in the d. Right Ventricle b. IUD
c. Rice with beef tapa and fried egg knee-to-chest position and administer oxygen. c. Diaphragm
d. Vegetable salad with shrimp and apples d. Assess the infant’s heart rate and rhythm. d. Oral contraceptive pills
33. What is the best position for James
24. Chrisa was prescribed take home medications – 28. Now that Dan has severe cyanosis, you anticipate a. Supine 38. Fertility awareness involves detecting when a
inhaled Salmeterol and Fluticasone – for long-term which drug to give? b. Prone woman is fertile so she can use periods of abstinence
management of asthma. Which of the following a. Indomethacin c. Semi Fowler during that time. If a couple is having coitus, when is it
responses show that her mother understands the b. Diclofenac d. Left sims lying safe to avoid pregnancy?
correct order of taking these medications? c. Celecoxib a. When the woman’s basal body temperature rises a
a. Chrisa inhales the Salmeterol first and then waits 5 d. Alprostadil 34. Patients with increased ICP have a high risk of full degree
minutes before inhaling the Fluticasone. seizures. If the pediatric patient starts to actively seize. b. When the woman’s cervical mucus becomes copious,
b. Chrisa inhales the Fluticasone first and then waits 5 29. You are now caring for Cess, a 3 year-old patient What is the very first thing the nurse should do? thin, watery, and transparent
minutes before inhaling the Salmeterol. who has a large atrial septal defect who is waiting for a. Maintain a patent airway c. When the woman is breastfeeding for 7 months
surgery. She is currently experiencing complications b. Ensure the surroundings are safe
d. When the woman is two days away from her a. Purging b. Progesterone
menstrual period 43. The parents then ask the nurse what medications b. Anorexia nervosa c. FSH
will be used to help cure the child. What is the nurse's c. Binge-eating disorder d. LH
39. Hannah, 24 years old and G2P2, has a copper- best response? d. Bulimia nervosa
based IUD. What do you expect to include in the a. "The doctor will order methylphenidate (Ritalin)." 54. Sheila typically has a menstrual cycle of 34 days.
teaching plan? b. "Treatment for autism focuses on behavior change, 48. A 17 year old gymnast is admitted to the hospital She tells you she had sexual intercourse on days 8, 10,
a. Amenorrhea is a common side effect of IUDs. not medications." due to weight loss and dehydration secondary to 15, and 20 of her last cycle. Which is the day on which
b. Severe cramping may occur when the IUD is inserted. c. "Dennis will be admitted to a psychiatric facility for starvation. Which of the following nursing diagnoses she most likely conceived?
c. She needs to have it changed yearly. medications." will be given priority for the client? a. Day 8
d. IUDs are more costly than other forms of d. “Dennis will be given medication at an outpatient a. Altered self-image b. Day 10
contraception. clinic." b. Fluid volume deficit c. Day 15
c. Altered nutrition: less than body requirements d. Day 20
40. The Anti-Violence Against Women and their 44. You are also caring for Karen, 6 years old, who is d. Ineffective coping
Children Act of 2004 seeks to address the prevalence of being evaluated for attention-deficit/hyperactivity 55. Cleo is already in her menopausal age. Because of
violence against women and their children (VAWC) by disorder. Which assessment finding (persisting more 49. In the management of patients with bulimia, the the hormonal changes present in women at
their intimate partners like their husband or ex- than 6 months) is consistent with the DSM-5 diagnostic following nursing interventions will promote a menopause, what condition should you educate her
husband, live-in partner or former live-in criteria for hyperactivity and impulsivity? therapeutic relationship, except: about prevention because it is most common?
partner, boyfriend/girlfriend or ex-boyfriend/ex- a. Often has difficulty waiting for turn in line or activity a. Establish an atmosphere of trust a. Breast cancer
girlfriend, dating partner or former dating partner. b. Often has difficulty organizing tasks and activities b. Discuss their eating behavior. b. Fractures
Which of the following situations is not included in c. Often loses things necessary for tasks or activities c. Help patients identify feelings associated with binge- c. Diabetes
psychological abuse? d. Often fails to finish schoolwork, chores, or duties purge behavior d. Anemia
a. When angry, Kim destroys their household property d. Teach patient about bulimia nervosa
in front of Casie. 45. If diagnosed with ADHD, which of the following 56. Kara, 35 years old, is currently pregnant with 23
b. Martin threatens Kyla that he will inflict physical drugs do you expect to be prescribed to improve focus, 50. The client with anorexia nervosa is improving if: weeks AOG. She has 2 children, the first one is 10 years
harm on himself if she leaves her. and decrease impulsivity and hyperactive behavior, a. She eats meals in the dining room. old and the other one is 8 years old. The first one was
c. After filing for annulment, Oli deprives Keith of which are the three hallmark ADHD symptoms? b. She attends ward activities. delivered at 38 weeks AOG and the second one was
custody and access to their twins. a. Aripiprazole (Abilify) c. She has a more realistic self concept. delivered at 35 weeks AOG. She got pregnant when she
d. After their break up, Ben continuously stalks Hailey b. Risperidone (Risperdal) d. She gains weight. was 19 years old, but miscarried at 2 months AOG. She
everytime she goes out. c. Citalopram (Celexa) also had a false pregnancy when she was 23 years old.
d. Methylphenidate (Ritalin) Situation What is her obstetric score?
Situation Caring for Childbearing and Childrearing Families a. G5T1P1A2L2
Cognitive delay may be present in some pediatric Situation b. G4T2P0A1L2
patients. Nurses have an important role in ensuring that Teenagers are one of the most vulnerable groups in 51. A couple went to you to ask regarding conception. c. G5T1P1A1L2
they maintain safety and are assisted towards developing eating disorders. The following questions When assessing the adequacy of sperm for conception d. G4T1P1A1L2
development. apply. to occur, which of the following is the most useful
criterion?
41. The nurse is evaluating Dennis, a 3-year-old child 46. Who among the following patients has the highest a. Sperm count 57. What is the psychological task of Kara?
with a developmental delay. Which assessment finding risk of developing an eating disorder? b. Sperm motility a. Accept the pregnancy
would indicate that this child might have a type of a. Diana, 3 years old, who is a picky eater. c. Sperm maturity b. Accept the baby
autistic spectrum disorder? b. Celine, 15 years old, who has been bullied for being d. Semen volume c. Prepare for parenthood
a. The child does not enjoy playing frequently with the larger than her peers. d. Anticipate baby’s needs
same toy. c. Kiana, 18 years old, who is a working student. 52. A woman states that she finds it hard to conceive
b. The child goes to bed without a nighttime routine. d. Carlo, 14 years old, has been training in basketball because she has a condition in which the entire uterus 58. Which of the following physiological changes is
c. The child is using echolalia. since he was 8 years old. is tipped far forward. What term is applicable to this not expected in Kara?
d. The child enjoys imaginative play. positional deviation of the uterus? a. Quickening
47. The nurse is assessing a client who is obese and a. Anteversion b. Anemia
42. Which of the following statements by Dennis’s reports eating to the point of discomfort at least twice a b. Retroversion c. Urinary frequency
parents determines that a goal of treatment has been week for the past year. The client denies the use of c. Anteflexion d. Leukorrhea
met? laxatives, self-induced vomiting, ipecac syrup, or d. Retroflexion
a. Dennis now eats with a fork. enemas and reports feeling unable to control the 59. Kara then tells you that she is experiencing leg
b. Dennis repeats questions asked to him. behavior. The client feels embarrassed and has stopped 53. What hormone is active early in the cycle and is cramps and is awakened by the cramps at night. To
c. Dennis sits still for 1 minute this morning. going out with friends. Which eating disorder should the responsible for maturation of the ovum? provide relief from the leg cramps, you advise the client
d. All of the above nurse suspect? a. Estrogen to:
a. Dorsiflex the foot while extending the knee when the d. Heparin-associated thrombosis and a. LOA Situation
cramps occur thrombocytopenia (HATT) b. ROP Shane, a 35-year-old female, is currently 16 weeks AOG.
b. Dorsiflex the foot while flexing the knee when the c. LOP She reported the following health history:
cramps occur 65. Because of the blood dyscrasias that may occur in d. ROA G5P4T3P1A0L4, BMI 29, diagnosis of hypertension and
c. Plantar flex the foot while flexing the knee when the Celine, which assessment finding is least likely to be depression, and family history of Type 2 diabetes.
cramps occur associated with this hematological condition? 71. When doing perineal care in preparation for
d. Plantar flex the foot while extending the knee when a. Swelling of the calf in one leg delivery, you should observe the following, except: 76. Which among the following risk factors increases
the cramps occur. b. Prolonged clotting times a. Use up-down technique with one stroke Shane’s risk of developing gestational diabetes mellitus?
c. Decreased platelet count b. Clean from the mons veneris to the anus Select all that apply
60. Upon checking Kara’s fundal height, she d. Petechiae, oozing from injection sites, and hematuria c. Use mild soap and warm water a. 35-years-old
complains of feeling dizzy and lightheaded. Her skin is d. Paint the inner thighs going towards the perineal area b. 16 weeks pregnant
also pale and moist. What is your initial response? Situation c. G5P4T3P1A0L4
a. Assess Kara’s blood pressure and pulse Intrapartum and Postpartum care 72. You transported a client in labor to the delivery d. BMI 29
b. Have Kara breathe into a paper bag room and prepared her for a cesarean delivery. Upon e. Hypertension
c. Raise Kara’s legs 66. You are caring for a client in the active phases of transferring her to the delivery room table, you place f. Depression
d. Turn Kara on her side. labor. The nurse is assessing the fetal patterns and the client in what position? g. Family history of Type 2 diabetes
notes a late deceleration on the monitor strip. The most a. Trendelenburg’s position with the legs in stirrups
Situation appropriate nursing action is to: b. Semi-Fowler position with a pillow under the knees a. e,b,c,g
Celine arrives at the emergency department (ED) with a. Place the mother in the supine position c. Prone position with the legs separated and elevated b. a,c,d,g
abruptio placentae at 34 weeks’ gestation. The b. Document the findings and continue to monitor the d. Supine position with a wedge under the right hip c. b,d,g
following questions apply. fetal patterns d. a.c.d
c. Administer oxygen via face mask 73. You are instructing a postpartum mother
61. Upon reviewing her history, which of the d. Increase the rate of pitocin IV infusion regarding lochia and the amount of expected lochia 77. Which test below is used to diagnose a patient
following data puts her at most risk for abruptio drainage. You instruct the mother that the normal with this condition?
placentae? 67. You are reviewing the record of a client in the amount of lochia may vary but should never exceed the a. 1 hour glucose tolerance test
a. Diagnosis of hypertension 3 years ago labor room and note that the fetus is at (-1) station. You need for: b. 24 hour urine collection
b. 34 weeks AOG document that the fetal presenting part is: a. One peripad per day c. Hemoglobin A1C
c. Exercising during pregnancy a. 1 cm above ischial spine b. Three peripads per day d. 3 hour glucose tolerance test
d. Eating more than recommended caloric intake b. 1 in below coccyx c. Eight peripads per day
c. 1 fingerbreadth below symphysis pubis d. Twelve peripads per day 78. fter being diagnosed with GDM, Shane asks you
62. What should you assess in Celine? d. 1 in below iliac crest how it can affect her baby. As the nurse, you know that
a. Bright red, painless vaginal bleeding 74. Five hours after a difficult labor and birth, a her baby is at risk for? Select all that apply.
b. Concealed or external dark red bleeding 68. During stage 3 of labor, you note a gush of blood primiparous woman refuses to hold and feed her baby, a. Hyperglycemia
c. Palpable fetal outline and that the uterus changes shape from an oval shape stating that she is too tired and just wants to sleep. You b. Hypoglycemia
d. Soft and nontender abdomen to globular shape. This indicates what? should: c. Respiratory distress
a. Postpartum hemorrhage a. Tell the woman she can rest after she feeds her baby d. Hyperbilirubinemia
63. Assessment showed that Celine is experiencing b. Imminent delivery of the baby b. Recognize this as a behavior of the taking-hold stage e. Hyperthermia
moderate vaginal bleeding. Based on these findings, you c. Signs of placental separation c. Record the behavior as ineffective maternal-newborn
would prepare the client for: d. Answers B and C attachment 79. Fortunately, Shane’s pregnancy was healthy and
a. Complete bed rest for the remainder of the 69. Upon assessment, you got the following findings: d. Take the baby back to the nursery, reassuring the she delivered a healthy baby boy. However, upon check
pregnancy 2 perineal pads highly saturated with blood within 1.5 woman that her rest is a priority at this time up, you observe several interactions between Shane
b. Delivery of the fetus hours postpartum, PR = 90 bpm, fundus soft and and her new son. What statement would you
c. Strict monitoring of intake and output boundaries not well defined. The appropriate nursing 75. Methergine or pitocin was prescribed for your investigate further to identify a possible maladaptive
d. The need for weekly monitoring of coagulation diagnosis is: client with postpartum hemorrhage. However, upon behavior regarding parent-infant attachment?
studies until the time of delivery a. Normal blood loss reading the patient’s history, you decided to contact her a. “My baby is hungry all the time. He wants to
b. Blood volume deficiency healthcare provider before medication administration breastfeed every hour.”
64. Based on your knowledge regarding abruptio c. Inadequate tissue perfusion related to hemorrhage because you noted that she has which of the following b. “This is my fifth baby. I’m not sure how we can
placentae, Celine is at risk for which of the following d. Hemorrhage secondary to uterine atony conditions? handle our finances for all of our children, but my
blood dyscrasias? a. Peripheral vascular disease husband will take care of it.”
a. Thrombocytopenia 70. During an internal examination, the nurse b. Hypothyroidism c. “This will be my last pregnancy. I don’t think it’s
b. Idiopathic thrombocytopenic purpura (ITP) palpated the posterior fontanel to be at the left side of c. Hypotension healthy for me to breastfeed and lose more sleep in the
c. Disseminated intravascular coagulation (DIC) the mother at the upper quadrant. The interpretation is d. Type 1 diabetes future.”
that the position of the fetus is:
d. “I don’t like looking at my baby’s eyes. I only be investigated further as a possible condition of the determine position, you palpated that the baby lies c. “I need to wear a supportive bra to relieve the
remember the pain of delivering him and now thinking fetus? transversely. What landmark is this? discomfort.”
how he will affect our finances.” a. Down syndrome a. Occiput d. “I need to stop breastfeeding until this condition
b. Trisomy 21 b. Mentum resolves.”
80. depressed at home with my new baby?” What are c. Open body defect c. Sacrum
the helpful guidelines you may give? Select all that d. Decreased lung surfactant d. Acromion 95. What psychosocial component is developed when
apply. a mother responds to a baby’s need to feed?
i. Plan a balanced program of nutrition, exercise, 85. During a prenatal visit a patient tells you her last Situation a. Trust
and sleep. menstrual period was August 28, 2016. Based on the Breastfeeding is an important activity among mothers. b. Love
ii. Stay with your baby all the time. Naegele's Rule, when is the estimated due date of her The following questions apply. c. Respect
iii. Share your feelings with a support person. baby? d. Warmth
iv. Avoid doing other stuff aside from baby care. a. July 4, 2017 91. According to the Philippine Milk Code of 1986 or
v. Do not try to be perfect. b. June 3, 2017 EO 51, which among the following is recommended to Situation
a. i, ii, iv c. June 4, 2017 protect and promote breastfeeding? Ethical principles apply in all nursing care. You are
b. i, iii, v d. July 1, 2016 a. Advertisement of breastmilk substitutes caring for a Sarah, a pregnant mother who belongs to
c. i, iii, iv, v b. Infant formula manufacturers giving mothers free an Indigenous People group.
d. i, ii, iii, iv, v 86. During a prenatal visit a patient tells you her last breastmilk substitutes
menstrual period was January 30, 2016. Based on the c. Including infant formula milk in relief goods 96. Cultural awareness is an in-depth self-
Situation Naegele's Rule, when is the estimated due date of her d. Teaching mothers in the community how to collect examination of one’s:
Fetal development and pregnancy baby? breast milk a. Background, recognizing biases and prejudices
a. November 6, 2016 b. Social, cultural, and biophysical factors
81. A prenatal client at 35 weeks gestation is b. October 25. 2017 92. The nurse assesses the mother of a newborn who c. Engagement in cross-cultural interactions
scheduled for an amniocentesis to determine fetal lung c. November 4, 2017 is breastfeeding. Which assessment finding indicates d. Motivation and commitment to caring
maturity. The nurse expects the lecithin/sphingomyelin d. October 22, 2016 ineffective breastfeeding of the newborn?
(L/S) ratio to be: a. The newborn’s mouth grasps the mother’s nipple 97. Which factor is least significant during assessment
a. 0.5:1 87. Your patient is 24 weeks pregnant, and you’re with the tongue down. when gathering information about Sarah’s cultural
b. 1:1 measuring the fundal height. Which finding below is a b. The newborn swallows audibly, spontaneously, and practices?
c. 2:1 normal measurement for this patient? frequently. a. Language, timing
d. 3:1 a. 16 cm c. The newborn smacks their lips loudly during b. Touch, eye contact
b. 28 cm breastfeeding. c. Biocultural needs
82. A client states that she had a spontaneous c. 26 cm d. The newborn turns to the breast when its cheek is d. Pain perception, management expectations
abortion 12 months ago. The client asks if her hormones d. 12 cm stroked with a nipple.
may have contributed to the loss of the pregnancy. The 98. If an ethical dilemma regarding care for Sarah
nurse’s response is based upon her knowledge of which 88. A patient is 40 weeks pregnant, and you find the 93. Which of the following is the primary predisposing arises, the solution to the dilemma requires negotiation
of the following facts? fundal height to be 4 cm below the xiphoid process. factor related to mastitis? among members of the healthcare team. Your point of
a. Implantation occurs when progesterone levels are Your next nursing action is to: a. Epidemic infection from nosocomial sources localizing view is valuable because:
low. a. Notify the MD immediately in the lactiferous glands and ducts a. Nurses have a legal license that encourages their
b. hCG reaches a maximum level at 4 weeks gestation. b. Chart this as a normal finding b. Endemic infection occurring randomly and localizing
c. Progesterone decreases the contractility of the c. Place the patient on continuous fetal monitoring in the peri glandular connective tissue presence during ethical discussions.
uterus. d. Assess the baby’s heart rate with a Doppler c. Temporary urinary retention due to decreased b. The principle of autonomy guides all participants to
d. Progesterone is only produced by the corpus luteum perception of the urge to avoid respect their own self-worth.
during pregnancy. 89. Starting 16 weeks AOG, a pregnant mother may d. Breast injury caused by overdistention, stasis, and c. Nurses develop a relationship to the client that is
secrete colostrum. The hormone _____ is responsible cracking of the nipples unique among all professional health care providers.
83. Amniocentesis was done on a pregnant client 16 for its production, while the hormone _____ is d. The nurse’s code of ethics recommends that a nurse
weeks AOG. You documented that the fetal urine is a responsible for its secretion. 94. You are providing instructions to a mother who be present at any ethical discussion about client care.
strong yellow. How would you interpret this? a. Prolactin, oxytocin has been diagnosed with mastitis. Which of the
a. Normal, fetal urine is dark yellow in color. b. Oxytocin, prolactin following statements, if made by the mother, indicates 99. While Sarah is in severe pain, you left her alone
b. Abnormal, this indicates blood incompatibility c. Estrogen, prolactin a need for further teaching? with the side rails down, and the bed in a high position.
c. Abnormal, this indicates presence of meconium d. Progesterone, estrogen a. “I need to take antibiotics, and I should begin to feel She then falls and developed bleeding. What law has
d. Abnormal, this indicates decreased alpha fetoprotein better in 24-48 hours.” been broken?
90. You are conducting Leopold’s Maneuver to a b. “I can use analgesics to assist in alleviating some of a. Assault
84. Upon checking for the amniotic fluid, you noted pregnant client. While doing the umbilical grip to the discomfort.” b. Battery
that the alpha fetoprotein (AFP) is too high. What can c. Negligence
d. Civil tort 5. What do you call situations in which the nurse is
aware of the correct course of action but institutional
100. Despite Sarah being economically
disadvantaged, you provided her quality care and gave
RECALLS 5 – NP3 constraints stand in the way of pursuing the correct
action?
Situation
her resources she needed just like other patients. What a. Moral distress
ursing practice is governed by ethical principles which
ethical principle is applicable in this situation? b. Moral dilemmas
serve as the basis for reasoning, actions, and decisions.
a. Beneficence c. Moral problems
b. Justice d. Moral uncertainty
1. Respect for autonomy requires individuals to honor
c. Non-maleficence
a patient’s right to self determination. This also involves
d. Autonomy 6. Cushing’s Syndrome is most likely to be developed
allowing patients and families to know all the relevant
in which of the following patients?
information and enabling them to weigh their options.
a. A patient with TB infection
What is the most important role that nurses have in
b. A patient with a pituitary gland tumor, causing
such situations?
increased ACTH secretion
a. Advocate
c. A patient who underwent adrenalectomy
b. Caregiver
d. A patient who is medicating with glucocorticoids for 6
c. Educator
weeks
d. Care manager
7. The following, except one, are clinical
2. What principle underlies a trusting relationship
manifestations of Cushing’s Disease?
between nurse and patient?
a. Buffalo hump
a. Beneficence
b. Hyperglycemia
b. Nonmaleficence
c. Skin hyperpigmentation
c. Veracity
d. Hirsutism
d. Justice
3. The principle of double effect may morally justify
8. A patient was admitted to ER, unconscious and
some actions that produce both good and evil effects.
with BP 70/40 mmHg. His wife reported that she has
Which of the following criteria must be fulfilled for an
been complaining about severe pain, increased
action to be considered moral based on the principle of
confusion, and lethargy. He said that he cannot recall
double effect?
when was the last time they bought her medication PO
i. The action itself is good or morally neutral.
Prednisone. What is the patient most likely
ii. The agent sincerely intends the good and not
experiencing?
the evil effect
a. Cushing’s Syndrome
iii. The evil effect is foreseen and intended.
b. Addisonian Crisis
iv. The good effect is not achieved by means of
c. Hashimoto’s Disease
the evil effect.
d. Thyroid Crisis
v. There is a proportionate or favorable balance
of good over evil.
9. What medication is expected to be started for the
a. i, ii, v
patient above?
b. i, iii, v
a. PO Prednisone
c. i, ii, iv, v
b. PO Declomycin
d. i, ii, iii, iv
c. IV Insulin
d. IV Solu-Cortef
4. What do you call situations in which a clear conflict
exists between two or more moral principles or
10. The patient has a take home medication which is
competing moral claims, where the nurse must choose
Prednisone. The following statements suggest effective
the lesser evil?
teaching, except?
a. Moral distress a. I will wear a Medic-Alert bracelet.
b. Moral dilemmas b. I will take this medication religiously at 8am
c. Moral problems everyday.
d. Moral uncertainty c. I will take this medication even when my symptoms
are gone.
d. I will stop taking the medication if I feel more sick.
21. What is the most common type of b. Occur only in malignant hypertension
Situation 16. The nurse is admitting a patient diagnosed with hyperthyroidism? c. Appear once the kidney has been irreversibly
Diabetes insipidus affects about 1 in 25,000 people in type 2 diabetes mellitus. The nurse should expect the a. Grave’s Disease damaged
the general population. Adults are more likely to following symptoms during an assessment, except: b. Hyperaldosteronism d. Appear when there is high risk of stroke
develop the condition, but it can occur at any age. a. Frequent bruising c. Hashimoto’s Disease
b. Dry mouth d. Adrenal Insufficiency 27. The patient was prescribed a new drug. Which
11. A patient arrives at the ER after a car accident c. Moist skin drug inhibits calcium ion influx into vascular smooth
with the following findings: BP 140/90 mmHg, PR 113 d. Tachycardia 22. All but one may be expected in a patient with muscle?
bpm, RR 20 cpm, T 37.5 deg C. After 48 hours, the hyperthyroidism. a. Atenolol
patient’s urine output increased to 180mL/hr and she is 17. Health teaching is important in nursing care for a. Diffuse pruritus b. Verapamil
extremely thirsty. Which laboratory findings support the patients with diabetes. The following guidelines are b. Exophthalmos c. Hydralazine
nurse’s suspicion of diabetes insipidus? helpful dietary recommendations for patients with DM, c. Cold intolerance d. Hydrochlorothiazide
a. Above-normal urine and serum osmolality levels except: d. Atrial fibrillation
b. Below-normal urine and serum osmolality levels a. Combine starchy foods with protein- and fat- 28. Non-pharmacological techniques can also help
c. Above-normal urine osmolality level, below-normal containing foods 23. A 40-year old woman returns from a subtotal lower blood pressure. All but one are examples of these
serum osmolality level b. Eat whole fruit instead of drinking juice thyroidectomy for the treatment of hyperthyroidism. techniques:
d. Below-normal urine osmolality level, above-normal c. Eat a light high-carbohydrate snack before exercising Upon assessment, the immediate priority that the nurse a. Smoking cessation
serum osmolality level d. Avoid eating breakfast should assess is: b. Taking multivitamins
a. Pain c. Dietary changes
12. Diabetes insipidus is characterized by: 18. Nurse Rico gave an intermediate-acting insulin to b. Neurological status d. Caffeine limitation
a. Decreased vasopressin and decreased kidney water patient A at 7PM. At 3AM, he noticed that the patient c. Fluid volume status
reabsorption was experiencing night sweats and his skin was cold to d. Respiratory distress 29. The patient is being discharged home on
b. Increased vasopressin and increased kidney water touch. When the patient woke up at 7AM, his glucose Hydrochlorothiazide (HCTZ) for treatment of
reabsorption level was 250 mg/dL. What is patient A most likely 24. What intervention for hyperthyroidism may be hypertension. Which of the following statements by the
c. Decreased insulin and increased serum glucose levels experiencing? delegated to the experienced certified nursing patient indicates they understood your discharge
d. Increased insulin and decreased serum glucose levels a. Dawn phenomenon assistant? teaching about this medication?
b. Somogyi effect a. Instruct the patient to report palpitations, dyspnea, a. I will make sure I consume foods high in potassium.
13. Which of the following interventions should not c. Insulin waning vertigo, or chest pain. b. I understand a dry cough is a common side effect
be included in the nursing care plan? d. Hypoglycemic crisis b. Check the apical pulse, blood pressure, and with this medication.
a. IV infusion of hypotonic saline solution temperature every 4 hours. c. I will only take this medication if my blood pressure is
b. Fluid restriction 19. Nurse Rico gave an intermediate-acting insulin to c. Draw blood for thyroid-stimulating hormone, T3, and high.
c. IM administration of ADH patient B at 8PM. At 2AM, his blood glucose is within his T4 levels. d. I will monitor my glucose levels closely because this
d. Skin care normal range. However, when the patient woke up at d. Explain the side effects of propylthiouracil (PTU) to medication may mask symptoms of hypoglycemia.
6am, his glucose level increased to 225 mg/dL. What is the patient.
14. Diabetes insipidus may be nephrogenic in form patient B most likely experiencing? 30. Which of the following patients does not have a
when there is failure in the renal tubules’ response to a. Dawn phenomenon 25. Lugol’s solution is given before a patient can risk factor for hypertension?
ADH. All but one may cause this condition: b. Somogyi effect undergo thyroidectomy. What is the rationale for this? a. A 25 year old male with a BMI of 35.
a. Hypokalemia c. Insulin waning a. Decrease the total basal metabolic rate. b. A 35 year old female with a total cholesterol level of
b. Hypercalcemia d. Hypoglycemic crisis b. Maintain the function of the parathyroid glands. 100.
c. Intake of declomycin c. Block the formation of thyroxine by the thyroid gland. c. A 68 year old male who reports smoking 2 packs of
d. Meningitis 20. What is the most appropriate nursing intervention d. Decrease the size and vascularity of the thyroid cigarettes a day.
for patient B? gland. d. A 40 year old female with a family history of
15. The patient with DI is given Desmopressin IV. a. Give a bedtime snack hypertension and diabetes
Which of the following nurse findings require b. Give the insulin at 10PM Situation
intervention? c. Increase insulin dose A 36-year old male went to the clinic for a check-up. He Situation
a. Clear breath sounds d. This is normal in patients with DM has been diagnosed with hypertension type I 4 years Maria is a 65-year old female diagnosed with Chronic
b. Decreased urine output ago. Kidney Disease, Type 2 diabetes mellitus, uncontrolled
c. Sodium level of 136 mEq/L Situation hypertension and acute dermatitis. She was admitted to
d. Potassium level of 5.4 mEq/L Thyroid disorders can range from a small, harmless 26. The patient says to the nurse, “I really don’t know the hospital with chief complaint of “ang lakas ng
goiter (enlarged gland) that needs no treatment, to life- why I need to come back regularly. I feel fine and kabog ng dibdib ko.”
Situation threatening cancer. haven’t had any symptoms.” The nurse would explain to
Diabetes mellitus is one of the most common lifestyle the client that symptoms of hypertension:
diseases worldwide. a. Are often absent
31. Maria’s potassium level was noted to be 5.7 36. Denver, 30 year-old male, is admitted with Situation Iran: an experimental study” by Farahani et al. (2021),
mmol/L. Her doctor ordered an ECG reading. Which of decreased level of consciousness and uncontrolled atrial Nursing research plays a big role in upholding and what is the independent variable?
the following is not expected? fibrillation. His pulse rate and blood pressure has continuously advancing the nursing practice. Being a a. Supportive home care program
a. Peaked T wave increased. The patient’s medication history includes researcher is an important role of nurses to improve b. Caregiver burden
b. Prominent U wave vitamin D supplements and calcium. What type of their knowledge and skills in patient care and to c. Stroke patients in Iran
c. Prolonged PR interval stroke is this patient at MOST risk for? contribute in ensuring evidence-based practice. d. An experimental study
d. Wide QRS a. Ischemic thrombosis
b. Ischemic embolism 41. Nurse Gia is conducting a grounded theory 47. What is the dependent variable in #46?
32. Maria was also identified to have low serum c. Hemorrhagic research. Which of the following is most accurate a. Supportive home care program
sodium level. Nursing intervention for a patient with a d. Ischemic stenosis regarding the grounded-theory method? b. Caregiver burden
diagnosis of hyponatremia includes all of the following a. An etic perspective is used during data collection. c. Stroke patients in Iran
except: 37. MRI shows damage to the cerebellum. What b. It is a process of constructing human experience. d. An experimental study
a. Assessing for symptoms of nausea and malaise. assessment findings should you expect to find? c. Secondary sources are sometimes used.
b. Encouraging the intake of low-sodium liquids such as a. Vision problems d. It is an inductive approach. 48. Mamo et al. (2019) conducted the study,
coffee or tea. b. Balance impairment “Determinants of poor glycemic control among adult
c. Monitoring neurologic status c. Language difficulty 42. Respondents of a research should always be patients with type 2 diabetes mellitus in Jimma
d. Restricting tap water intake. d. Impaired short-term memory protected. Which of the following rights of individual University Medical Center, Jimma zone, south west
participants must be ensured when the researcher Ethiopia: a case control study”. The aim of this study is
33. Upon assessment, you noticed that Maria has a 38. Another patient, Tokyo, has been admitted with cannot link the information given by the respondent to identify the determinants of poor glycemic control at
pitting edema on both of her lower legs. Her urine right side brain damage due to stroke. What can you from the source of the information? the diabetes clinic of the Jimma University Medical
output is 15-20mL/hr, dark amber in color. Her recent expect to observe in the patient? Select all that apply: a. Confidentiality Center from April 01 to June 30/2017. What is the other
GFR was 23 mL/min. What stage of chronic kidney i. Right side hemiplegia b. Anonymity term for this case control study?
disease is this known as? ii. Confusion on date, time, and place c. Virility a. Retrospective study
a. Stage 2 iii. Aphasia d. Volunteerism b. Prospective study
b. Stage 3 iv. Unilateral neglect c. Cohort study
c. Stage 4 v, Aware of limitations 43. Nurse Via integrates the findings of the multiple d. Cross-sectional study
d. Stage 5 vi. Impulsive quantitative research studies on nonpharmacological
34. Which medication ordered by the physician will vii. Short attention span interventions for patients with ischemic stroke. What 49. Sample is regarded as a subset of:
help treat the patient's uncontrolled hypertension along viii. Agraphia type of research method is this? a. Data
with providing a protective mechanism to the kidneys? a. i, iii, iv, vi, vii a. Meta-synthesis b. Set
a. Metoprolol b. ii, iii, iv, viii b. Meta-analysis c. Distribution
b. Amlodipine c. ii, iv, vi, vii c. Secondary analysis d. Population
c. Verapamil d. All of the above d. Systematic analysis
d. Lisinopril 50. Nurse Paul suggests for their community research
39. Tokyo was prescribed Mannitol for increased ICP. 44. What process should Nurse Bon use to obtain a to conduct data collection about everyone or everything
35. Maria is now experiencing extreme pruritus and Which statement is INCORRECT about this medication? 10-year-old child’s agreement for participation in the in group or population and has the advantage of
has several areas of crystallized white deposits on the a. Mannitol will remove water from the brain and place study, “Eating practices of hospitalized children under accuracy and detail. What is this called?
skin. As the nurse, you explain that this is due to it in the blood to be removed from the body. 12 years of age”? a. Census
excessive amounts of what substance found in the b. Mannitol will cause water and electrolyte a. Assent b. Survey
blood? reabsorption in the renal tubules. b. Consent of a minor c. Probability sampling
a. Calcium c. When a patient receives Mannitol, the nurse must c. Minor agreement d. Cluster sampling
b. Urea monitor the patient for both fluid volume overload and d. None of the above
c. Phosphate depletion. Situation
d. Erythropoietin d. Mannitol is not for patients who are experiencing 45. Once Nurse Bon collected all necessary data via Nurse Lisa, a new registered nurse has been oriented
anuria. interviews, what technique can he use to analyze these about many ethical and moral issues that she may
Situation data? encounter while in practice. The following question
The mortality from stroke during the last 10 years has 40. What assessment finding requires immediate a. Chi-Square test applies.
an average of 63,804 deaths per year. Despite the intervention if found while Tokyo is receiving Mannitol? b. Measures of variation
COVID pandemic, the recorded annual Philippine stroke a. Crackles c. Content analysis 51. Nurse Lisa was asked by a fourth year nursing
death increased from 64,381 in 2020 to 68,180 in 2021 b. BP 110/80 d. Pearson r correlation learner regarding the BEST practical guide they can use
(Collantes et al., 2022). c. ICP of 10 mmHg if confronted with ethical dilemmas. Nurse Lisa should
d. Dry mouth and thirst 46. In the study “The effect of a supportive home care inform the nursing learner about the ________.
program on caregiver burden with stroke patients in a. Florence nightingale oath
b. RA 9173 about to begin a 6-month course of therapy with myxedema coma. Immediate nursing intervention is
c. Code of ethics for nurses Situation isoniazid. Nurse Al should plan to teach the client to warranted with which assessment data?
d. Deontological theory Mr. Ricci Rivero, 25 years old, a college instructor was take which action? a. Serum blood glucose level of 60-mg/dl
admitted to the Mariano Marcos Memorial Hospital and a. Increase intake of Swiss or aged cheese b. Pulse oximeter reading of 90%
52. Nurse Lisa emphasized to the nursing learners that Medical Center due to difficulty of breathing 30 minutes b. Drink Gin bilog in small amounts only c. Telemetry reading showing sinus bradycardia
a nurse should fully support the patient’s rights prior to admission. He also revealed that he has been c. Avoid vitamin supplements during therapy. d. The client is lethargic and sleeps all the time.
throughout their personal stay when working toward suffering from blood-streaked productive cough d. Report yellow eyes or skin immediately. Situation
achieving desirable outcomes for them. This ethical accompanied by dull and aching chest with coughing for Mrs. Ehe, 42 years old was diagnosed with Grave’s
principle is known as? more than four weeks now. He was diagnosed with Situation disease.
a. Teleology pulmonary tuberculosis 2 years ago. Ms. Cynthia went to the hospital for a follow-up check
b. Accountability up after her hospitalization due to exacerbated 66. The nurse is expected to asses from Mrs. Ehe with
c. Advocacy 56. Nurse Al is assigned to patient Ricci and should hypothyroidism. which of the following sign?
d. Beneficence expect the lungs to be with _______ in the chest x-ray a. Anorexia
result? 61. Nurse Jen is assigned to patient Cynthia. The nurse b. Tachycardia
53. Mr. Mosi has presented in the emergency a. Pulmonary infiltrates should expect which signs/symptoms Ms. Cynthia may c. Cold skin
department complaining of substernal chest pain. He b. Multinodular infiltrates exhibit? d. Sleep problems
has a history of diabetes and hypertension. The c. Crackles a. Complains of extreme fatigue and hair loss
physician considered Mr. Mosi to be at risk of CAD that d. Stridor b. Exophthalmos and complains of nervousness 67. Prophylthiouracil (PTU) is prescribed to Mrs. Ehe.
needs surgery. What PRIORITY nursing actions should c. Complaints of profuuse sweating and flushed skin. The nurse should inform the client to report ______
be done by the members of the health team in this 57. Nurse Al administered oxygen and had carried out d. Tetany and complaints of stiffness of hands. immediately.
situation? all the doctors order. Nurse Al should be aware that a. Painful, excessive menstruation
a. Inform Mr. Mosi of his health status and discuss documentation of the care rendered should be done 62. Nurse Jen should be aware that the preferred b. Sore throat
possible interventions, then let him decide. __? medication for treating hypothyroidism is? c. Constipation
b. Proceed immediately to surgery after confirming the a. As soon as possible a. Propylthiouracil d. Increased urine output
presence of clot in his heart’s artery b. After the shift b. Propranolol
c. Consult his wife and let her decide on behalf of Mr. c. Every four hours c. Synthroid 68. Another patient was diagnosed with
Mosi. d. every 8 hours d. Lithium hyperthyroidism and was prescribed with methimazole.
d. Refer the patient to the ethics committee. Select all nursing considerations that
58. Mr. Rivero was discharged after 5 days in the 63. Patient Cynthia had discussed the onset of feelings are NOT associated with this medication.
54. Which statement would best explain the role of the hospital. Nurse Al carried out the referral forms to San of sadness and depression due to her illness to the a. Administer methimazole with food
nurse when planning care for a culturally diverse NicoIlas Municipal Health Office. Nurse Alwin should nurse, the nurse should inform the client that these b. Place the client on a low-calorie, low- protein
population? The nurse will plan care to: expect that the health information of patient Brent is in feelings are: diet
a. Focus only on the needs of the client, ignoring the what form of the FHSIS? a. normal and part of having a chronic illness. c. Assess the client for unexplained bruising and
nurse’s beliefs and practices a. Target Client List b. Temporary effects of thyroid replacement therapy bleeding
b. Include care that is culturally congruent with the staff b. Treatment record and will diminish once homeostasis is achieved d. Instruct the client to report side and adverse
from predetermined criteria c. Tally or Reporting Forms c. Permanent effects of thyroid replacement therapy effects such as sore throat, fever, or headaches
c. Blend the values of the nurse that are for the good of d. Output Records and will not diminish over time e. Use special radioactive precautions when
the client and minimize the client’s individual values d. Most likely related to low thyroid hormone levels and handling the client’s urine for the first 24 hours
and beliefs during care 59. Nurse Al has conducted discharge teaching with Mr. will improve with treatment following initial administration.
d. Provide care while aware of one’s own bias, focusing Rivero regarding his medications. Nurse Al determines 64. Which findings indicate the presence of side effects a. abcd
on the client’s individual needs rather than the staff’s that Mr. Rivero has understood the information if he associated with levothyroxine? Select all the apply. b. bcde
practices makes which statement? a. Weight loss c. acd
a. “Hindi na ako makakapunta ng Sogo para kumain sa b. Bradycardia d. be
55. Nurses agree to be advocates for their patients. susunod na anim na buwan” c. Mild heat intolerance
Practice of advocacy calls for the nurse to: b. “Nararapat na ipagpatuloy ko ang aking gamutan sa d. Constipation 69. Which signs/symptoms should make the nurse
a. Assess the client’s point of view and prepare to isang buwan” e. Insomnia suspect the client is experiencing a thyroid storm?
articulate this point of view. c. “Makakapagtrabaho na ako kapag ang aking sputum a. a,b,c a. Extreme tachycardia and hypothermia
B. Document all clinical changes in the medical record in culture ay negatibo da baktirya” b. a,c,d,e b. Hypotension and tachycardia
a timely manner. d. “Hindi na ako makakahawa pagkatapos ng 2-3 c. b,c,d,e c. Decreased respirations and hypoxia
C. Work to understand the law as it applies to the linggong gamutan” d. a,c,e d. Hyperpyrexia and extreme tachycardia
client’s clinical condition.
D. Seek out the nursing supervisor in conflicting 60. Another patient was admitted to the MMMH-MC 65. After several weeks from the follow-up check-up
situations who was recently diagnosed with pulmonary TB. He is patient Jen was admitted to the ICU diagnosed with
70. In advance cases of hyperthyroidism, the diagnosis 76. Nurse Jeremy prepares the blood to be transfused. dehydrated and is suffering anemia secondary to a. ELISA
is made on the basis of the symptoms and which of the As a prudent nurse verification of the blood typing and pneumonia. After checking the patency of their IV lines b. Soberano
following laboratory result? cross matching result should be done to prevent any and vital signs, what should the nurse do next c. Western Blot
a. Increase in serum TSH transfusion reaction. Should nurse Jeremy fails to verify a. Call for both client’s blood transfusions at the same d. Rumpel Leads test
b. Decreased free T4 the blood typing and cross-matching result and has time.
c. Decrease in radioactive iodine uptake transfused an incompatible ABO- and Rh- blood, what b. Call for and hang the first client’s blood transfusion 86. The nurse is admitting a 28-year-old college
d. Decreased in serum TSH type of reaction should nurse Jeremy expect? c. Ask another nurse to call for and hang the blood for instructor with a diagnosis of rule-out anemia. The
a. Febrile reaction the second client client has undergone a gastric bypass surgery for
Situation b. Hemolytic reaction d. Ask another nurse to verify the compatibility of both obesity 3 years ago. Current assessment findings
Nurse John has just passed the board exam and is c. Allergic reaction units at the same time. include height of 5’5”; weight 75 kg; P110, R27, and BP
planning to work in a tertiary hospital in his province. d. Overacting reaction Situation 104/66; pale mucous membranes and dyspnea on
Nurse John should be encouraged to abide with the Ms. Serena, 25 years old, has been recently diagnosed exertion. The nurse should suspect the client to have
laws that govern the nursing practice in the Philippines. 77. In the situation above, nurse Jeremy should assess with HIV. She has been so distressed about her which type of anemia?
the patient with what signs/symptoms? diagnosis. a. Sickle cell anemia
71. What is the PRC Modernization Act of 2000? a. Fever and chills b. Folic acid anemia
a. RA 9881 b. Flushing and urticaria 81. A year after her diagnosis, she went back to the c. Iron deficiency anemia
b. RA 8981 c. Dyspnea and stridor laboratory for her follow up CD4 counting. Her CD4 d. Vitamin B12 deficiency
c. RA 1989 d. Dyspnea and hypotension count has been noted to be 120 cell/mm^3. The nurse
d. RA 9198 should be aware that which symptom/s should be 87. A client is admitted with full-thickness burns and
78. Nurse Jeremy should be well aware of the most present aside from the laboratory report would indicate may be developing DIC. Which findings would support
72. All successful candidates in the examination are important nursing considerations when caring for a her condition has already progressed to AIDS? the diagnosis of DIC
required to take an oath of profession before ______. client who is receiving 2 units of packed RBCs. Select all a. Malaise,fever, anorexia, and weight loss a. Sudden onset of chest pain and frothy sputum
a. Any PRC Official that apply b. Leukopenia b. Foul-smelling, concentrated urine
b. The chief of hospital a. Verify that the ABO and Rh of the 2 units are c. Fatigue c. Oozing blood from the IV catheter site
c. The Baranggay tanod of their respective Baranggay the same. d. Mucopurulent greenish-yellowish cough d. A reddened, inflamed central line catheter site.
d. Any authorized government official b. Infuse a unit of PRBCs in less than 4 hours
c. Stop the transfusion if a reaction occurs, but 82. The nurse is assigned to care for Ms. Serene who 88. A patient asks the primary nurse, “How does
73. The principal basis of the code of ethics for nurses is keep the line open has developed a cutaneous Kaposi’s sarcoma. The nurse someone get hemophilia A?” Which statement would
______. d. Take vitals signs every 15 minutes while the is about to give the patient a bath, change the bed be the primary nurse’s best response?
a. Local Government Code of the Philippines unit is transfusing linens. The lesions are open and drains a small amount a. “ it is an inherited A-linked recessive disorder
b. Code of Good Governance e. Inspect the blood bag for leaks, abnormal of serous fluid. What PPE should the nurse use to bath b. “It is an inherited X-linked recessive disorder”
c. RA 9173 color, and clots. the patient c. “The person is born with hemophilia A”
d. ANA Code of Ethics for Nurses f. Use a gauge 22 catheter for optimal flow of a a. Gloves only d. “The mother carries the gene and gives it to the son”
blood transfusion. b. Gown and gloves for both procedures
74. According to the Code of ethics for Nurses, the a. a,c,e c. Gown, gloves and a mask for both procedures 89. The student nurse asks the nurse, “What is sickle
hallmark of nursing accountability focuses on _____. b. d,e d. Gown and gloves to change the bed linens,and gloves cell anemia?” Which statement by the nurse would be
a. Accurate documentation c. b,c,e only for the bath the best answer to the student’s question?
b. Safe nursing practice d. a,b,c,d,e,f a. “Bakit nag tatanong ang beshy ko?”
c. Quality and excellence 83. In the first few months of her diagnosis, Ms. Serena b. “It is a congenital disease of the blood in which the
d. Welfare of patients 79. Patient Dee had already received 25 ml of packed was encouraged to continue taking zidovudine (AZT). blood does not clot”
RBCs has complained of low back pain and pruritus. The expected outcome of AZT is to: c. “The client has decreased synovial fluid that causes
75. As a well-informed registered nurse, nurse John After stopping the infusion, nurse Jeremy should take a. Destroy the virus joint pain.”
should earn ___ CPD units to renew his license every what action next? b. Slow replication of the virus d. “The blood becomes thick when the client is deprived
three years a. Administer prescribed antihistamine and aspirin c. Enhance the body’s antibody reaction of oxygen”
a. 13 b. Administer diuretics d. Neutralize toxins produced by the virus
b. 14 c. Administer vasopressors 90. Which is a potential complication that occurs
c. 15 d. Collect blood and urine samples and send to the lab 84. How long is the window period of HIV? specifically to a male client diagnosed with sickle cell
d. 16 a. 1-3 months anemia during a sickle cell crisis?
80. Nurse Jeremy has two middle-aged clients who b. 4-5 months a. ejaculation
Situation were prescribed with blood transfusion of PRBCs at the c. 6-7 months b. Inflammation of the clitoris`
Patient Dee has been ordered to receive a blood same time. The first client’s blood pressure dropped d. 8-9 months c. Priapism
transfusion after a hemorrhagic episode. from preoperative value of 120/80 mmHg to a d. Hypertensive crisis
postoperative value of 100/50. The second client is 85. What is the confirmatory test for HIV?
Situation b. Ask the AP to tell the client in pain that medication a. Easy to test data for reliability and validity
Quality nursing practice impact patient care, patient will be administered as soon as the bed bath is b. Facilitates data gathering
outcomes, patient and clinical satisfaction, and resource complete c. Less time consuming than interview and observation
utilization. The nursing management plays a vital role in c. Finish the bed bath and then administer the pain d. Respondents may provide socially acceptable
achieving a quality nursing practice. medication to the other client answers
d. Cover the client, raise the side rails, tell the client that
91. Nurse Laida is the chief nurse of Mariano Marcos you will come back shortly, and administer the pain
Memorial Hospital and Medical Center. The ER medication to the other client.
Department has been receiving feedback regarding long
waiting times for treatment. Nurse Laida has formed a 95. Nurse Laida is preparing a training program to
committee to resolve the issue. The leader of the introduce quality management. Which information
committee should first consider what task to solve the should Nurse Laida include?
problem? a. Staff satisfaction surveys will be used to track the
a. Reassess the emergency waiting times. effectiveness of changes
b. Consult the director of operations for possible b. The purpose is to improve the experience of patient
solutions care, population, and values.
c. Collect baseline data to determine if a problem exists c. Staff will plan in silos
d. A continuous quality improvement team should be d. Programs will be staff focused.
established to define the desired outcome
Situation
92. A staff nurse in the pediatric department had a Nursing research is considered essential to the
medication error and had immediately reported to the achievement of high-quality patient care and outcomes.
unit supervisor. The pediatric department is known to
promote quality nursing practice. Which response 96. Which of the following is a method of non-
should the staff expect from the supervisor? probability sampling?
a. "Why are you reporting that to me without a. Cluster sampling
completing an incident report?" b. Snowball sampling
b. "You need to report directly to the Chief Nursing c. Simple random sampling
Officer." d. stratified random sampling
c. "We have a blame-free environment so you can
report errors without fearing punishment." 97.The data on the family’s number of children is
d. "You have reported this to me, so you do not need to appropriate for what level of measurement?
file an incident report." a. Interval
93. The nurse has received the assignment for the day b. Ordinal
shift. Which client should the nurse plan to care c. Ratio
for first after making initial rounds and assessment d. Nominal
a. A client with a white blood cell count of 14, 000 mm3
(14x109/L) and a temperature of 38.4oC 98. A nurse researcher wants to study the response of
b. A client who is ambulatory demonstrating steady gait patients who suffer from dysrhythmia to pacemaker.
c. A postoperative client who has just received an opioid The appropriate research design would be?
pain medication. a. Experimental research design
d. A client scheduled for physical therapy for the first b. Descriptive-correlational
crutch-walking session. c. Comparative descriptive
d. Correlational
94. The nurse is giving a bed bath to an assigned client
when an assistive personnel (AP) enters the client’s 99. Which of the following are qualitative data sources?
room and tells the nurse that another assigned client is a. Interview and observation.
in pain and needs pain medication. Which is the most b. Primary sources and secondary sources
appropriate nursing action? c. Books and journals
a. Ask the AP to find out when the last pain medication d. Questionnaires and survey
was given to the client
100. All of the following are advantages of using
questionnaires EXCEPT:
d. Restore client's cardiac rhythm 12. The type of research design that does not
RECALLS 5 – NP4 7. When preparing the epinephrine injection from an
manipulate independent variable is:
a. Experimental design
19. In an attempt to reduce the cases of typhoid fever in
the community, SDG No. ___ will guide the programs
Situation
ampule, the nurse initially: b. Quasi-experimental design that will be created for this initiative.
Nurse Jairo is admitting a patient with DM type I. In
a. Taps the ampule at the top to allow fluid to flow to c. Non-experimental design a. 4
caring for patients with DM type I, it is imperative that
the base of the ampule d. Quantitative design b. 5
nurses must be knowledgeable about the different
b. Checks expiration date of the medication ampule c. 6
types of insulin and their respective duration to
c. Removes needle cap of syringe and pulls plunger to 13. This research topic has the potential to contribute d. 7
promote safe patient care.
expel air to nursing because it seeks to:
d. Breaks the neck of the ampule with a gauze wrapped a. Include new modalities of care 20. Which assessment finding would warrant the nurse
1. What is the duration of action of Humulin N
around it b. Resolve a clinical problem to report to the physician?
a. 18-24 hours
c. Clarify an ambiguous modality of care a. Bleeding from the rectum
b. 20-22 hours
8. Mrs. Simon is obese. When administering a d. Enhance client care b. Fever
c. 21-23 hours
subcutaneous injection to an obese patient. It is best for c. Greenish stool
d. 16-20 hours
the nurse to: d. Muscle pain
2. What is the duration of action of Lantus? a. Inject needle at a 15 degree angle over the stretched 14. Martha does review of related literature for the
a. >24 hours skin of the client purpose of: 21. The board of nursing is composed of how many
b. 5-12 hours b. Pinch skin at the injection site and use airlock a. Determine statistical treatment of data research members?
c. 5-7 hours technique b. Orientation to what is already known or unknown a. 7
d. 20-24 hours c. Pull skin of patient down to administer the drug in a Z c. To identify if problem can be replicated b. 6
track d. Answering the research question c. 5
3. What is the duration of action of Humulin R? d. Spread skin or pinch at the injection site and inject d. 4
a. 5-7 hours needle at a 45-90 degree angle 15. Client's rights should be protected when doing
b. 6-12 hours research using human subjects. Martha identifies these 22. The __________ is the appointing person of any
c. 8-9 hours 9. When preparing for a subcutaneous injection, the rights as follows EXCEPT: vacancy in the Board of Nursing.
d. 1-2 hours proper size of syringe and needle would be: a. right of self-determination a. PNA
a. Syringe 3ml and needle gauge 21 to 23 b. right to compensation b. President of the Philippines
4. What is the duration of action of Apidra? b. Tuberculin syringe 1 ml with needle gauge 26 or 27 c. right of privacy c. Chairperson
a. 3-5 hours c. Syringe 2ml and needle gauge 22 d. right not to be harmed d. DOH secretary
b. 6-7 hours d. Syringe l-3ml and needle gauge 25 to 27
c. 6-9 hours Situation 23. Who nominates, chooses and ranks the
d. 6-8 hours 10. The rationale for giving medications through the Patient Nicole was admitted to the communicable recommendees for a position in the boars?
subcutaneous route is; disease unit due to typhoid fever. a. MCNAP
5. The nurse is administering the initial dose of a rapid- a. There are many alternative sites for subcutaneous b. PRC
acting insulin to a client with DM type I. The nurse injection 16. What is the usual range of incubation period of c. DOH
should asses the client for hypoglycemia within: b. Absorption time of the medicine is slower typhoid fever? d. Accredited professional organization
a. 3 hours c. There are less pain receptors in this area a. 1-3 weeks
b. 0.5 hours d. The medication can be injected while the client is in b. 7-14 days 24. The “term” of office for the board is ____ years?
c. 1 hours any position c. 2-3 weeks a. 6 years
d. 2 hours d. 4-7 weeks b. 9 years
Situation c. 10 years
Situation The use of massage and meditation to help decrease d. 3 years
Mr. Ibarra is assigned to the triage area and while on stress and pain have been strongly recommended based 17. What are the principal vehicles of typhoid fever?
duty, he assesses the condition of Mrs. Simon who on documented testimonials. a. Food and water 25. The board examination should be in accordance
came in with asthma. She has difficulty breathing and b. Mosquito with RA 8981 or the _____.
her respiratory rate is 40 per minute. Mr. Ibarra is asked 11. Martha wants to do a study on the topic. "Effects of c. Rad a. PRC Modernization Act
to inject the client epinephrine 0.3mg subcutaneously. massage and meditation on stress and pain." The type d. Typhoon b. The Philippine Nursing Law
of research that best suits this topic is: c. CMO 15, series of 2017
6. The indication for epinephrine injection for Mrs. a. Applied research d. None of the above
18. What are the sources of infection?
Simon is to: b. Qualitative research a. Feces and urine
a. Reduce anaphylaxis c. Basic research Situation
b. Blood
b. Relieve hypersensitivity to allergen d. Quantitative research Frenzy, a 42 year old patient, is hospitalized with a
c. Semen
c. Relieve respiratory distress due to bronchial spasm diagnosis of AKI resulting from the administration of
d. Saliva of mosquito
gentamicin sulfate for a Pseudomonas aeruginosa b. Frothy, bloody sputum and consolidation 36. Which instruction should the nurse provide when a. Hold urine, as long as she can before emptying the
infection. Frenzy is acutely ill upon admission and c. Barrel chest and polycythemia discussing exercise? bladder to strengthen her sphincters muscles
experiencing an altered level of consciousness. d. Bronchovesicular lung sound and bradypnea a. Walk at least 30 minutes a day on flat surfaces b. If burning sensation is experienced while voiding,
b. Perform lightweight lifting three times a week drink pineapple-juice
26. The nurse notes that a patient who is retaining fluid 32. The client had a right-sided chest tube inserted 1 c. Recommend high-level aerobics daily c. After urination, wipe from anal area up towards the
had a 1-kg weight gain. Documentation should indicate hour ago for a pneumothorax. The nurse should d. Encourage the client to swim laps once a week pubis
that it is equivalent to about how many ml? implement which action if the water-seal compartment d. Tell client to empty the bladder at each voiding
a. 250 ml does not show any fluctuation? 37. The health-care provider prescribes an ACE inhibitor
b. 500 ml a. Order for a STAT chest x-ray for the client. Which statement is the best rationale for
c. 750 ml b. Increase the amount of wall suction administering this medication. 42. Mrs. Seva also tells the nurse that she is often
d. 1000 ml c. Check the tubing for kinks or clots a. ACE inhibitors prevent beta receptor stimulation in constipated. Because she is aging, what physical
d. Monitor the client’s pulse oximeter reading the heart changes predispose her to constipation?
27. In the case of Frenzy, what should the nurse b. This medication blocks the alpha receptors in the a. inhibition of the parasympathetic reflex
assigned to her do? Select all the apply. 33. Mrs. Simon refuses to take deep breaths because of vascular smooth muscle b. weakness of sphincter muscles of the anus
a. Elevate the head of the bed 30-45 degrees pain. She has had a right-sided chest tube for 6 hours c. ACE inhibitors prevent vasoconstriction and sodium c. loss of tone of the smooth muscles of the colon
b. Call the admitting physician for prescriptions now. Which intervention should the nurse implement? and water retention d. decreased ability to absorb fluids in the lower
c. Establish an IV access site a. Medicate the client and help the client take deep d. ACE inhibitors decrease blood pressure by relaxing intestines
d. Contact the hemodialysis unit breaths vascular smooth muscles
a. abcd b. Encourage the client to take shallow breaths to help 43.The nurse understands that one of these factors
b. bcde with the pain 38. Another patient is diagnosed with hypertension. Her contributes to constipation:
c. cdea c. Explain deep breaths do not have to be taken at this blood pressure is 135/85 mmHg. Using the blood a. excessive exercise
d. abce time pressure categories of the American Heart Association, b. high fiber diet
d. Tell the client that he could die if she refuse to take the client is considered to have c. no regular time for defecation daily
28. Which of the following is the most common initial deep breaths again a. Elevated d. prolonged use of laxatives
manifestation of acute renal failure? b. Stage 1 hypertension
a. Dysuria 34. The nurse who is caring for Mrs. Simon has seen c. Stage 2 hypertension 44.Mrs. Seva asked for instructions for skin care for her
b. Anuria that the chest tube is accidentally pulled out of the d. Stage mom mother who has urinary incontinence and is almost
c. Hematuria pleural space. Which action should the nurse implement always in bed. Your instruction would focus on
d. Oliguria first? 39. You are to measure the client's initial blood pressure prevention of skin irritation and breakdown by ____.
a. Notify the health care provider reading by doing all of the following EXCEPT a. Using thick diapers to absorb urine well
29. Patient Frenzy who is in acute renal failure has an b. Instruct the client to take slow shallow breaths until a. Take the blood pressure reading on both arms for b. Drying the skin with baby powder to prevent or mask
elevated blood urea nitrogen (BUN). What is the likely the tube is reinserted comparison the smell of ammonia
cause of this finding? c. Do nothing b. Listen to and identify the phases of Korotkoff's c. Thorough washing, rinsing and drying of skin area
a. Fluid retention d. Tape a dry occlusive dressing on three sides to the sounds that get wet with urine
b. Reduced renal blood flow insertion site c. Pump the cuff up to around 50 mmHg above the d. Making sure that linen are smooth and dry at all
c. Hemolysis of red blood cells point where the pulse is obliterated times
d. Below-normal metabolic rate 35. The nurse is presenting a class on chest tubes. d. Observe procedures for infection control
Which statement best describes a tension 45. Over the years, Mrs. Seva was confined to bed (1).
30. Patient Frenzy’s serum potassium level is elevated in pneumothorax? 40. diagnosed with hypertension about following a low- Upon assessment, the nurse notes that the patient
acute renal failure, and the nurse administers sodium a. A tension pneumothorax develops when an air-filled calorie, low-fat, low-sodium diet. Which of the following responds only to painful stimuli (2). The perineal is kept
polysterene sulfonate (Kayexalate). This drug acts to: bleb on the surface of the lung ruptures menu selections would best meet the client’s needs? moist by urine as dampness is detected every time the
a. Increase potassium excretion from the colon b. When a tension pneumothorax occurs, the air moves a. Mixed green salad with blue cheese dressings, patient is moved (1). In addition, she can not make even
b. Release hydrogen ions for sodium ions freely between the pleural space and the atmosphere crackers and cold cuts a slight change in her body position without assistance
c. Increase calcium absorption in the colon c. The injury allows air into the pleural space but b. Ham sandwich on rye bread an an orange (1) but can maintain a relatively good position in bed
d. Exchange sodium for potassium ions in the colon prevents it from escaping from the pleural space c. Baked chicken, an apple, and a slice of white bread most of the time but occasionally slides down (2). Her
d. A tension pneumothorax results from a puncture of d. Hot dogs, baked beans, and celery and carrot sticks daughter revealed that she rarely eats meal (2). The
Situation the pleura during a central line placement Situation nurse scores Mrs. Seva’s risk for pressure sore using the
Mrs. Simon is admitted to the emergency department Mrs. Seva, 32 years old, asks you about possible Braden Scale with?
with chest trauma following a car collision. Situation problems regarding her elimination now that she is in a. 7
Georgia, a 30 year old woman, is diagnosed as having the menopausal stage. b. 8
31. Which signs/symptoms indicate to the nurse the essential hypertension when serial blood pressure c. 9
presence of pneumothorax recordings show her average reading to be 170/100 41. Instruction on health promotion regarding urinary d. 10
a. Unequal lung expansion and dyspnea mmHg. elimination is important. Which would you include?
Situation Situation 61. Which of the following is true regarding the
Nurse Rivera witnesses a vehicular accident near the Situation Management of nurse practitioners is done by qualified treatment of head lice?
hospital where she works. She decides to get involved Nurse George has worked a lot in the slums of Tondo, nursing leaders who have had clinical experience and a. All household members and other close contacts
and help the victims of the accident. Manila and the life there is very... it's poor and it's very management experience. should be checked.
sad. And he had always taught himself to look for the b. Only the affected household member should be
46. Her priority nursing action would be to: beauty in it; to look in the beauty in the faces of the 56. An example of a management function of a nurse is: checked for head lice as the host may vary from person
a. Assess damage to property children, and to be grateful.Unfortunately, last week, a. Teaching patient do breathing and coughing exercises to person.
b. Assist in the police investigation since she is a witness Nurse George has witnessed a large fire that consumes b. Preparing for a surprise party for a client c. Some experts believe that siblings of the affected
c. Report the incident immediately to the local police the almost the houses and almost killed thousands of c. Performing nursing procedures for clients person should be checked as this can be caused by
authorities lives in Tondo, Manila. Nurse George was tasked to d. Directing and evaluating the staff nurses genetic predisposition.
d. Assess the extent of injuries incurred by the victims, classify the different types of burn according to depth in d. None of the above.
of the accident the emergency department. 57. Your head nurse in the unit believes that the staff
nurses are not capable of decision making so she 62. Aling Nena, one of the residents of Baranggay
51. A patient who has burned arms characterize as makes the decisions for everyone without consulting Kababaihan asks the nurse the most appropriate way to
47. Priority attention should be given to which of these shiny and moist with blisters that are painful. anybody. This type of leadership is: wash the clothes of her daughter with head lice 2 days
clients? a. 3rd degree burn a. Laissez faire leadership just before treatment is started. The best response of
a. Linda who shows severe anxiety due to trauma of the b. 2nd degree burn b. Democratic leadership the nurse would be?
accident c. 1st degree burn c. Autocratic leadership a. Machine wash and dried using the hot water and hot
b. Ryan who has chest injury, is pale and with difficulty d. 4th degree burn d. Managerial leadership air cycles
of breathing b. Do not wash yet because the head lice may fly and go
c. Noel who has lacerations on the arms with mild- 52. A patient whose burns are located at his back that 58. When the head nurse in your ward plots and to her
bleeding are pink and painful. approves your work schedules and directs your c. Wash only with bleach and then boil
d. Andy whose left ankle swelled and has some a. 1st degree burn work, she is demonstrating: d. Do nothing
abrasions b. 2nd degree burn a. Responsibility
c. . 3rd degree burn b. Delegation 63. Aling Nena was also advised to seal items that can
48. In the emergency room, Nurse Rivera is assigned to d. 4th degree burn c. Accountability not be laundered for ___ weeks in a plastic bag to
attend to the client with lacerations on the arms, while d. Authority prevent it from crawling to another person.
assessing the extent of the wound the nurse observes 53. A patient whose burns are located on his right thigh a. 3 weeks
that the wound is now starting to bleed profusely. The with blisters and weeps. 59. The following tasks can be safely delegated by a b. 4 weeks
most immediate nursing action would be to: nurse to a non-nurse health worker EXCEPT: c. 5 weeks
a. Apply antiseptic to prevent infection a. 1st degree burn a. Transfer a client from bed to chair d. 2 weeks
b. Clean the wound vigorously of contaminants b. 2nd degree burn b. Change IV infusions
c. Control and reduce bleeding of the wound c. 3rd degree burn c. Irrigation of a nasogastric tube 64. A pediculicide is prescribed to the daughter of Aling
d. Bandage the wound and elevate the arm d. 4th degree burn d. Take vital signs Nena. What important instruction should the nurse
impart to her?
49. The nurse applies pressure dressing on the bleeding 54. A patient whose burns are located on his left arm 60. You made a mistake in giving the medicine to the a. 1-3 days
site. This intervention is done to: complaining of no pain to which the burns are wrong client. You notify the client's doctor and write b. 1-4 days
a. Reduce the need to change dressing frequently characterized as blackish leathery burns. an incident report. You are demonstrating: c. 1-2 days
b. Allow the pus to surface faster a. 1st degree burn a. Responsibility d. 1-5 days
c. Protect the wound from micro organisms in the air b. 2nd degree burn b. Accountability
d. Promote hemostasis c. 3rd degree burn c. Authority 65. After 1-2 days of not re-washing the hair, what
d. 4th degree burn d. Autocracy instruction should the nurse give to aling nena
50. After the treatment, the client is sent home and regarding the combs and brushes use?
asked to come back for follow-up care. Your 55. The nurse is monitoring a child with burns during Situation a. Clean the combs and brushes with bleach
responsibilities when the client is to be discharged treatment for burn shock. Which assessment provides ediculosis is infestation with the human head-and-body b. Soak them in cidex
include the following EXCEPT the most accurate guide to determine the adequacy of louse, Pediculus humanus. There are two subspecies, c. Spray them with alcohol
a. Encouraging the client to go to the, outpatient clinic fluid resuscitation? the head louse (P. h. capitis) and the body louse (P. h. d. Soak them in hot water for 5-10 minutes
for follow up care a. Skin turgor humanus). In the Philippines, head lice picking is one of Situation
b. Accurate recording, of treatment done and b. Level of edema at burn site the most enjoyed activities of many unemployed Philippine nurses are encouraged to be well aware of
instructions given to client c. Adequacy of capillary filling women in the streets. Hence, nurses play a big role in the laws that affect their practice regardless of the set-
c. Instructing the client to see you after discharge for d. Amount of fluid tolerated in 24 hours ending this health, and at the same time societal up and setting they wish to practice with.
further assistance problem.
d. Providing instructions regarding wound care
66. What is RA 7160? 72. Which of these does NOT occur in a descriptive d. Focusing activity on the correction of identified b. A client with suspected gastric cancer who is on
a. Local Government Code behavior. nothing-by-nouth (NPO) status for tests.
b. PRC Modernization Act study? c. A client with PUD experiencing a sudden onset of
c. Anti-rabies law a. manipulation of variable 77. Nurse Kathlyn should be well aware of that which of acute stomach pain
d. Newborn screening law b. explanation of relationships between two or more the following statements is NOT true about d. A client who is requesting pain medication 2 days
phenomena performance appraisal? after surgery to repair a fracture jaw.
67. What is RA 10354? c. investigation of a phenomena in real life context a. Informing the staff about the specific impressions of
a. Anti-Rabies Law d. exploration of relationships between two or more their work help improve their performance. 82. Nurse Jessa received in her shift a patient who has
b. Newborn screening law phenomena b. A verbal appraisal is an acceptable substitute for a just had an upper GI endoscopy. The patient’s vital signs
c. The Responsible Parenthood and Reproductive Health written report are ordered to be taken every 30 minutes for 2 hours
Law 73. A research group studied the trend in junk food c. Patients are the best source of information regarding after the procedure. Nurse Jessa assigned a UAP to take
d. None of the above personnel appraisal. the vital signs. One hour later, the UAP reports the
consumption of a group of females over a 10-year d. The outcome of performance appraisal rests primarily client, who was previously afebrile, has developed a
68. What is RA 11223? period using data they collected annually. This is an with the staff. temperature of 101.8oF (38.8oC). What should the
a. Universal Health Care Act example of a/an: nurse do in response to these assessment data.
b. Data Privacy Act a. case study 78. There are times when Katherine evaluates her staff a. Promptly assess the client for potential perforation.
c. Responsible Parenthood and Reproductive Health b. evaluative study as she makes her daily rounds. Which of the following is b. Tell the assistant to change thermometers and retake
Law c. longitudinal study NOT a benefit of conducting an informal appraisal? the temperature
d. None of the above d. cross-sectional study a. The staff member is observed in natural setting. c. Reassess the patient’s temperature
b. Incidental confrontation and collaboration is allowed. d. Asks the UAP to provide the client a tepid sponge
69. What is RA 10152 c. The evaluation is focused on objective data bath as this is just an expected outcome
a. Mandatory Infants and Children Health Act 74. In the research "The effect of listening to a systematically.
b. Responsible Parenthood and Reproductive Health relaxation tape on the anxiety levels of preoperative d. The evaluation may provide valid information for 83. When nurse Jessa is obtaining a nursing history from
Law compilation of a formal report. a client with suspected gastric ulcer, which signs and
c. Civil Registry Law patients," the independent variable is: symptoms should the nurse assess? Select all that
d. None of the above a. anxiety levels 79. She conducts a 6-month performance review apply.
b. preoperative patients session with a staff member. Which of the following a. Epigastric pain at night
c. listening to a relaxation tape actions is appropriate? b. Relief of epigastric pain after eating
70. What is RA 9288? d. relaxation tape a. She asks another nurse to attest the session as a c. Vomiting
a. Mandatory Infants and Children Health Act witness. d. Weight loss
b. Civil Registry Law b. She informs the staff that she may ask another nurse e. Melena
c. Data Privacy Act 75. A well written plan would include the following,
to read the appraisal before the session is over. a. acd
d. Newborn Screening Act EXCEPT c. She tells the staff that the session is manager- b. cdb
a .Research problem centered. c. ade
Situation b. Review of Related Literature and d. The session is private between the two members. d. cde
Nursing research develops knowledge about health and Theoretical/Conceptual Framework
the promotion of health over the full lifespan, care of c. Methods and Procedures 80. She checks the documentary requirements for the 84. Nurse Jessa is preparing to teach a client with a
persons with health problems and disabilities, and d. Research Findings applicants for staff nurse position. Which one is NOT peptic ulcer about the diet that should be followed after
nursing actions to enhance the ability of individuals to necessary? discharge. Nurse Jessa should explain that the client
respond effectively to actual or potential health Situation a. Certificate of previous employment should eat which of the following?
problems. Nursing leadership plays a vital role in shaping b. Record of related learning experience (RLE) a. Bland foods
outcomes for healthcare organizations, personnel and c. Membership to accredited professional organization b. High-protein foods
patients. With much of the leadership workforce set to d. Professional identification card c. Any foods that are tolerated
71. With a title "Effects of aromatherapy on the stress retire in the near future, identifying factors that d. A glass of milk
levels of businesswomen in an urban community", the positively contribute to the development of leadership Situation
reader would know that this is a/an: in nurses is of utmost importance. Nurse Jessa has been assigned to provide care for four 85. A client is taking a Magnesium hydroxide for
a. descriptive research clients in her department. treatment of PUD. Which of the following statements
b. quantitative research 76. Kathlyn as a unit manager, knows that performance best indicated that the clients understands how to
c. applied research appraisal consists of all the following activities EXCEPT: 81. Which among the patients will Nurse Jessa prioritize correctly take the antacid?
d. basic research a. Setting specific standards and activities for individual for assessment? a. ”I’ll take my antacids before I take my other
performance. a. A client awaiting surgery for a hiatal hernia repair at medications.”
b. Using agency standards as a guide. 11 AM. b. “I need to decrease my intake of fluids so that the
c. Determine areas of strength and weaknesses effects of the antacid will not be diluted.”
c. “I’ll take my antacid depending on the frequency of Situation Situation 100. Part of standards of care has to do with the use of
my pain” Mr. Woo is a 58-year-old Asian man who is scheduled Your director of nursing wants to improve the quality of restraints. Which of the following statements is NOT
d. “It Is best for me to take my antacid 1 to 3 hours after for permanent pacemaker insertion treatment for a health care offered in the hospital. As a staff nurse in true?
meal.” tachydysrhythmia that does not respond to medication that hospital you know that this entails quality a. Doctor's order for restraints should be signed within
therapy. He is scheduled for an endocardial implant. assurance programs. 24 hours
Situation b. Remove and reapply restraints every two hours
Nursing research has been a growing trend in the 21st 91. Nurse Beejay is reading the cardiac monitor and 96. A legislative enactment that serves as a defense to c. Check client's pulse, blood pressure and circulation
century nursing practice. notes that the patient’s heart rhythm is extremely malpractice is the Good Samaritan statute. The every four hours
irregular and there are no discernible P waves. The following statements are correct, except: d. Offer food and toileting every two hours
86. Nurse Jeremy checks if his instruments meet the ventricular rate is 90 beats per minute, and the patient a. It protects health care provides from civil liability that
criteria for evaluation. Which of the following criteria is hemodynamically stable. The nurse realizes that the may be incurred in stopping to render aid at the scene
refers to the consistency or the ability to yield the same patient is suffering from: of an accident.
response upon its repeated administration? a. Atrial fibrillation b. It also applies to hospital care given to a client as long
a. Validity b. Atrial flutter it is of an emergency nature
b. Reliability c. Atrial flutter with rapid ventricular response c. Health care professionals may still be sued by an
c. Sensitivity d. Junctional escape rhythm injured victim for gross negligence.
d. Objectivity d. Health care provides should not charge the patient
92. Nurse Jerry should understand that in a third-degree during an emergency if they want to be covered by the
87. Which criteria refer to the ability of the instrument AV block: statute.
to detect fine differences among the subjects being a. Every P wave is conducted to the ventricles.
studied? b. Some P waves are conducted to the ventricles 97. Standards of nursing practice serve as guide for:
a. Sensitivity c. None of the P waves are conducted to the ventricles a. Nursing practice in the different fields of nursing
b. Reliability d. The PR interval is prolonged b. Proper nursing approaches and techniques
c. Validity c. Safe nursing care and management
d. Objectivity 93. The nurse is attempting to determine the ventricular d. Evaluation of nursing cared rendered
rate and rhythm of a patient’s telemetry strip. What
88. Which of the following terms refer to the degree to should the nurse examine to determine this part of the 98. You are taking care of critically ill client and the
which an instrument measures what it is supposed to analysis? doctor in charge calls to order a DNR (do not
be measure? a. PP interval resuscitate) for the client. Which of the following is the
a. Validity b. QT interval appropriate action when getting DNR order over the
b. Reliability c. RR interval phone?
c. Meaningfulness d. TP interval a. Have the registered nurse, family spokesperson,
d. Sensitivity nurse supervisor and doctor sign
94. The nurse notes on the cardiac monitor that the PR b. Have two nurses validate the phone order, both
89. Nurse Jeremy is interested to learn more about interval is very irregular and that there are more P nurses sign the order and the doctor should sign his
transcultural nursing because he is assigned at the waves than QRS complexes. PP interval and RR interval order within 24 hours.
family suites where most patients come from different are regular. The nurse should suspect that the patient is c. Have the registered nurse, family and doctor sign the
cultures and countries. Which of the following designs is suffereing from: order
appropriate for this study? a. Third-degree atrioventricular block d. Have 1 nurse take the order and sign it and have the
a. Grounded theory b. Second-degree atrioventricular block doctor sign it within 24 hours
b. Ethnography c. Sinus tachycardia
c. Case study d. Sinus bradycardia 99. To ensure the client safety before starting blood
d. Phenomenology transfusion the following are needed before the
95. Which among the following statements correctly procedure can be done EXCEPT:
describes cardioversion? a. take baseline vital signs
90. He plans to use a Likert Scale to determine a. Cardioversion is the immediate and unsynchronized b. blood should be warmed to room temperature for 30
a. degree of agreement and disagreement delivery of current. minutes before blood transfusion is administered
b. compliance to expected standards b. Cardioversion is the delivery of the electrical current c. have two nurses verify client identification, blood
c. level of satisfaction synchronized with the patient’s electrical events type, unit number and expiration date of blood
d. degree of acceptance c. Either a or b d. get a consent signed for blood transfusion
d. None of the above
d. None of the above a. “Chiz, who? 15. The nurse should be aware that which of the
RECALLS 5 – NP5 5. How long would it take for a diagnosis of manic
b. “Tell me about Chiz”
c. “Tell me about you and Chiz”
following is the priority assessment for crisis?
a. Defense mechanism of the person
Situation
episode or mania be made? d. “That’s a good Chiz-mis” b. Financial stability
Jimot come to the mental health clinic saying that he
a. 4 weeks c. Perception of the patient to the crisis along with the
feels so down and lacking in energy with “loss of
b. 2 weeks 11. Which of the following is a concrete message? presence of support system and coping mechanism
interest in everything.” He tells the nurse that he
c. 3 weeks a. “Get this our of here.” d. None of the above
received some samples of a new medication from his
d. 1 week b. “When is she coming home?”
psychiatrist last week to relieve his depression.
c. “They said it is too early to get in.” 16. A 16-year-old who is being seen by the crisis nurse
6. Which of following described Bipolar type II? d. “Help me put this pile of books on Jeremy’s desk.” after making several superficial cuts on her wrist states
1. Nurse Gemma is aware that Jimot has a history of
a. Recurrent depressive episodes with at least one that all her friends are siding with her ex-boyfriend and
bipolar I disorder with hospitalization for a significant
hypomanic episode. Situation won’t talk to her anymore. She says she knows that the
manic episode. With this knowledge, the nurse would
b. Manic episodes with at least one depressive episode Typhoon Egay has recently struck the northern islands relationship is over, but “If I can’t have him, no one else
draw special concern regarding which category of psych
c. Alternating cycles between periods of mania, normal of the Philippines leaving vast damages to the will.” Which of the following client problems takes the
medications?
mood, depression, normal mood, mania, and so forth. properties. highest priority?
a. Atypical antipsychotics
d. None of the above a. Risk for other-directed violence
b. Mood stabilizers/antimanics
12. The nurse working at the site of a severe flood sees b. Situational low self-esteem
c. Antianxiety agents (benzodiazepines)
Situation Janang, standing in knee-deep water, staring at empty c. Risk for suicide
d. SSRI
Therapeutic communication is an interpersonal lot. Janang told the nurse, “Masamang panaginip lang d. Risk-prone health behavior
interaction between the nurse and the client during lahat ng ito. Bukas magigising akong nandyan pa ang
2. Nurse Gemma has observed Jimot who is hyperactive
which the nurse focuses on the client’s specific needs to bahay ko.” Which of the following crisis intervention Situation
and intrusive sitting very close to Jerlyn, a female
promote an effective exchange of information. strategies are most needed at this time? Select that The nurse is preparing a client with schizophrenia who
patient with his arm around her shoulders. The nurse
apply. has a history of command hallucinations for discharge
hears the male client cracking sexually explicit joke.
7. Client: “I had an accident” a. Ask the client about any physical injuries she may by providing instructions on interventions for managing
Nurse Gemma approaches Jimot and asks him to walk
Nurse: “Tell me about your accident” have. hallucinations and anxiety.
down the hallway. Which of the following statements
This is an example of which therapeutic communication b. Determine if any of her family are injured or missing.
by the nurse Gemma should be beneficial to the
technique? c. Allow the client to talk about her fears, anger, and 17. Which statement in response to these instructions
patient?
a. Making observations other feelings suggests to the nurse that the client has a need for
a. “Ayaw ni Jerlyn na nasa tabi mo siya dahil sa
b. Offering self d. Tell her that groups are being formed at the shelter additional information?
pananalita mo”
c. General lead for flood survivors a. “My medication will help my anxious feelings.”
b. “ Ang pag-sasabi ng mga malalaswang bagay at pag
d. Reflection e. Refer her to the shelter for dry clothes and food b. “I’ll go to support group and talk about what I am
hipo sa iba ay hindi pinahihintulutan dito”
f. Assess her for risk of suicide and other signs of feeling.”
c. “Kailangan mong maging maingat sa mga sinasabi mo
8. “Earlier today you said you were concerned that your decompensation c. “When I have command hallucinations, I’ll call a
sa ibang tao”
son was still upset with you. When I stopped by your a. abcd friend for help.”
d. “Kinakailangan mo ng pumunta sa iyong kwarto”
room about an hour ago, you and your son seemed b. bdef d. “I need to get enough sleep and eat well to help
relaxed and smiling as you spoke to each other. How did c. adef prevent feeling anxious.”
3. Another young adult client is diagnosed with bipolar
things go between the two of you?” This is an example d. abcf
disorder. He has been religiously taking his medications
of which therapeutic communication technique? 18. Which of the following group of symptoms are
and has managed the disorder effectively. One day, the
a. Encouraging comparison 13. Nurse Jan is assessing a client who has just considered positive symptoms?
client suddenly becomes manic. The nurse reviews the
b. Consensual validation experienced a crisis due to typhoon Egay. Nurse Jan a. Hallucination, Delusions, and Apathy
client’s medication record. Which among the following
c. Accepting should first assess this client for which of the following b. Asociality, Catatonia, and Flat affect
medications should the nurse expect to have
d. General lead behaviors? c. Inattention, avolition, and apathy
contributed to the development of his manic state?
a. Effective problem solving d. Perseveration, hallucination, and bizarre behavior
a. Amitriptyline
9. “How does Heart Evangelista make you upset?” is a b. Level of anxiety
b. Prednisone
non-therapeutic communication technique because it c. Attention Span 19. Neurochemical studies have consistently
c. Gabapentin
a. Gives a literal response d. Help-seeking demonstrated alterations in the neurotransmitter
d. Buspirone
b. Indicates an external source of the emotion systems of the brain in people with schizophrenia.
c. Interprets what the client is saying 14. Which of the following typify a situational crisis? Which of the following neurotransmitter is not
4. Which of the following drug is often used in
d. Is just another stereotyped comment a. Natural catastrophe implicated by several studies to have been associated
conjunction with mood stabilizers or anti-depressants
b. Death of a loved one with schizophrenia?
to treat bipolar disorder?
10. what is the best way for the nurse to ask the client c. menarche a. GABA
a. Ziprasidone
to describe her relationship with Chiz? d. Marriage b. Dopamine
b. Aripiprazole
c. Serotonin
c. Either A or B
d. Norepinephrine c. “Being assaulted is traumatic; in time the anxiety will a. “Therapy has been a very good thing for me since the
25. The nurse is caring for the client with a major lessen, and you’ll feel more in control.” hurricane ruined things.”
20. The primary medical treatment for schizophrenia is depressive disorder. Which nursing problem should be d. “By using the skills you’re learning, the goal for you is b. “I’m ready and able to move on with my life in spite
psychopharmacology. The firs-generation antipsychotics priority? to feel better or be back to normal in about 6 weeks.” of all that has happened.”
target which manifestation of schizophrenia? a. Powerlessness c. “Nothing can happen to me that is worse than what
a. Hallucinations b. Attempted suicide 30. The client presents to the ED reporting that he was I’ve been through already.”
b. Avolition c. Anticipatory grieving sexually assaulted by several men he met at a local bar. d. “I’ve learned a lot about myself since agreeing to
c. Alogia d. Disturbed sleep pattern Which action should the nurse plan to include when attend crisis therapy sessions.”
d. Alopecia preparing to assess the client?
26. The nurse is interviewing the client at a mental a. Ask the client if he had been drinking alcohol 34. The nurse in the ED is assessing the client who was
21. One of the side effects of antipsychotic medication health clinic who recently attempted suicide and excessively. injured in a car accident. The nurse considers that the
is neuroleptic malignant syndrome. What should be the continues to report active suicidal ideation. Which care b. Call the male nurse on duty to assume the care of this client may have psychogenic amnesia when the client is
nursing intervention should this occur? setting is most appropriate for this client? client. unable to recall any personal information. Which
a. Stop all antipsychotic medications; notify the a. An acute care hospital unit c. Do the interview in the same way as for other sexual statement that reflects the nurse’s critical thinking
physician b. An inpatient mental health unit assaults. about psychogenic amnesia is correct?
b. Administer medications as ordered c. An outpatient mental health clinic d. Ask whether the client resisted any of the sexual a. Psycho genie amnesia is a long—lasting condition.
c. Assess for effectiveness d. A community detoxification center advancements. b. Psychogenic amnesia is seen more often in men than
d. None of the above women.
Situation 31. The 10—year-old who was sexually abused by a c. Psycho genie amnesia is categorized with memory
Situation The recently discharged veteran who served in active family member experiences flashbacks of a loss and dementia.
Nurse John is aware that he has a crucial role in combat reports symptoms of recurring intrusive disagreement with that adult and the resulting sexual d. Psycho genie amnesia symptoms include wandering
managing clients with anxiety-related disorders. thoughts, insomnia, and hyper vigilance. assault. Which suggestion should the nurse make to the and disorientation.
parents in order to help minimize this reaction?
22. The client reports becoming involved with 27. Which question would be most helpful in a. Have the child avoid arguments with adults until this 35. Which among the following options correctly
legislation that promotes gun safety after the death of establishing a diagnosis? reaction is unlearned. describe debriefing
the child by accidental shooting. Which defense a. “Do you find yourself falling asleep while working?” b. Ask the HCP to prescribe a medication to minimize a. Participants are asked about their emotional
mechanism is the client exhibiting? b. “Are you also having nightmares when you do sleep?" the child’s aggressiveness. reactions to the incident, what symptoms they may be
a. Denial c. “Your hair seems thin. Are you also pulling at your c. Adults in your family should learn to recognize and experiencing and other psychological medications.
b. Sublimation hair?” diffuse arguments effectively. b. Is a process by which the person receives education
c. Identification d. “Have you ever been diagnosed with obsessive d. You and your child should regularly discuss bad about recognition of stress reactions and management
d. Intellectualization compulsive disorder?” memories to decrease their effect. strategies for handling stress.
c. Either A or B
23. The client reports becoming physically ill with 28. The nurse is caring for a victim of sexual assault Situations d. None of the above
frequent crying episodes, intense feelings of brought to the ED by a roommate. How should the The Philippines is home to many destructive typhoons.
worthlessness, and loss of appetite on the anniversary nurse respond when the client begins to angrily insist 36. The 10—year-old who was sexually abused by a
of the death of the client’s spouse. The client reports upon reporting the details of the assault? 32. The client’s home was destroyed by a major flood. family member experiences flashbacks of a
that this has occurred for the last 5 years- What should a. ask the roommate to sit with the client until the The client is attending a support group and says, “I will disagreement with that adult and the resulting sexual
be the nurse’s focus when counseling the client? examination can be resumed. rebuild my home as good as new and be back in it in a assault. Which suggestion should the nurse make to the
a. Anticipatory grief b. Redirect the client to the physical tasks related to few months.” What should be the nurse’s initial parents in order to help minimize this reaction?
b. Uncomplicated grief securing any existing evidence. response? a. Have the child avoid arguments with adults until this
c. Delayed grief reaction c. Encourage the client to use deep breathing a. “That’s a very ambitious plan to undertake at this reaction is unlearned.
d. Distorted grief reaction techniques to regain emotional control. time.” b. Ask the HCP to prescribe a medication to minimize
d. Listen quietly as the client expresses the anger and b. “I’m proud of your resiliency and willingness to start the child’s aggressiveness.
24. The client is being discharged after hospitalization rage currently being experienced over.” c. Adults in your family should learn to recognize and
for a suicide attempt. Which question asked by the c. “Have you given thought to what may happen if it diffuse arguments effectively.
nurse assesses the learned prevention and future 29. The young adult after being robbed is attending floods again?” d. You and your child should regularly discuss bad
coping strategies of the client? counseling sessions to address anxiety issues. What is d. “Can you tell me how many months you think memories to decrease their effect.
a. “How did you try to kill yourself?” the nurse’s best response when the client asks, “When rebuilding will take?”
b. “Why did you think life wasn’t worth living?” will things get better for me?” Situation
c. “What skills can you utilize if you experience a. “These types of crises are self-limiting, and usually 33. The client is being treated after surviving a major The client is hospitalized after sustaining a head injury
problems again?” things are better in 4 to 6 weeks.” hurricane that took the lives of many neighbors. Which and a fractured wrist from a fall. The client admits to
d. “Do you have the phone number of the suicide b. “Try not to worry; it is best for you to think about the statement by the client provides the nurse with the drinking alcohol in moderation several times per week.
prevention center?” future and not focus on the past.” [best evidence that therapy has been successful?
37. Which assessment finding should the nurse 41. The hospitalized client has a history of weekly d. At the time scheduled b. Drowsy or confused state following a seizure
associate with early alcohol withdrawal? moderate alcohol use. Which symptoms assessed by c. Severe itching of the eyes from an allergic reaction
a. Agitation the nurse indicate that the client may be experiencing 46. The nurse has asked the nursing assistant to d. Abnormal sensations including tingling of the skin
b. Somnolence alcohol withdrawal? Select all that apply. ambulate a client with Parkinson’s disease. The nurse
c. Slightly elevated BP a. Agitation observes the nursing assistant pulling on the client’s 51. The nurse asks the male client with epilepsy if he
d. Delirium tremens (DTs) b. Hypotension arms to get the client to walk forward. The nurse has auras with his seizures. The client says, “I don’t
c. Tachycardia should: know what you mean. What are auras?” Which
38. The nurse is caring for the client who is 2 days d. Hallucinations a. Praise the nursing assistant as this is appropriate. statement by the nurse would be the best response?
postadmission to a medical unit and has a long history e. Tongue tremor b. Explain how to overcome a freezing gait by telling the a. “Some people have a warning that the seizure is
of heavy alcohol abuse. The nurse should monitor for a. bcde client to march in place. about to start.”
which acute complications related to alcohol abuse? b. abcd c. Assist the NA with getting the client back in bed. b. “Auras occur when you are physically and
Select all that apply c. abcde d. Give the client a muscle relaxant as studies have psychologically exhausted.”
a. Seizures d. acde proved that this is effective in this situation. c. “You’re concerned that you do not have auras before
b. Pancreatitis your seizures?”
c. GI bleeding Situation Situation d. “Auras fight for her friend in the bar.”
d. Exophthalmos Nurse Mira is assigned to care for a patient with The male client is sitting in the chair and his entire body
e. Delirium tremens Parkinson’s Disease. is rigid with his arms and legs contracting and relaxing. Situation
a. edcb The client is not aware of what is going on and is making A nurse is caring for several patients who are suffering
b. abcd 42. Nurse Mira has admitted a patient with PD with a guttural sounds. from meningitis.
c. abce fever and patchy infiltrates in the lung fields on the
d. bcde chest x-ray. Which clinical manifestations of PD would 47. Which action should the nurse implement first? 52. The wife of the client diagnosed with septic
explain these assessment data? a. Push aside any furniture meningitis asks the nurse, “I am so scared. What is
39. The female client tells the nurse, “I usually have a a. Maskliek facies and shuffling gait. b. Place the client on his side meningitis?” Which statement would be the most
few drinks after work, but I always limit it to three. I’m b. Difficulty swallowing and immobility. c. Assess the client’s v/s appropriate response by the nurse?
not risking becoming addicted, am I?” What is the c. Pill rolling of fingers and flat affect d. Ease the client to the floor a. “There is bleeding into his brain causing irritation of
nurse’s best response? d. Lack of arm swing and bradykinesia the meninges.”
a. “There is no harm in social drinking as long as you 48. The client who just had a three minute seizure has b. “A virus has infected the brain and meninges, causing
know your limits and you are not driving while 43. The client diagnosed with PD is being discharged on no apparent injuries and is oriented to name, place, and inflammation.”
intoxicated.” carbidopa/levodopa (Sinemet), an antiparkinsonian time but is lethargic and just wants to sleep. Which c. “This is a bacterial infection of the tissues that cover
b. “As long as you don’t have any social problems drug. Which statement is the scientific rationale for intervention should the nurse implement? the brain and spinal cord.”
associated with your use of alcohol, you do not need to combining these medication? d. “This is an inflammation of the brain parenchyma
be concerned.” a. There will be fewer side effects with this combination a. Perform a complete neurological assessment caused by a mosquito bite.”
c. “If you are concerned about the frequency and the than with carbidopa alone. b. Awaken the client every 30 minutes
number of drinks consumed, then you might be b. Dopamine D requires the presence of both of these c. Turn the client to the side and allow the client to 53. The client is at risk for septic emboli after being
developing a dependency.” medications to work. sleep diagnosed with meningococcal meningitis. Which action
d. “Three drinks a day or seven drinks in a week is high- c. Carbidopa makes more levodopa available to the d. Interview the client to find out what caused the by the nurse directly addresses this risk?
risk drinking for women. You seem concerned that you brain. seizure a. Monitoring vital signs and oxygen saturation levels
might have an alcohol dependency.” d. Carbidopa crosses the BBB to treat Parkinson’s hourly
disease. 49. The nurse is teaching the client who is scheduled For b. Planning to give meningocoeeal polysaccharide
40. The nurse is preparing to administer thiamine an outpatient EEG. Which instruction should the nurse vaccine
(vitamin B,) to the client receiving treatment for alcohol 44. Which is a common cognitive problem associated include? c. Assessing neurological function with the Glasgow
dependence. Which statement best describes the with Parkinson’s disease? a. Remove all hairpins before coming in for the EEG test. Coma Scale q2h
rationale for the use of thiamine? a. Emotional lability b. Avoid eating or drinking at least 6 hours prior to the d. Completing a thorough vascular assessment of all
a. Thiamine improves the absorption of other essential b. Depression test. extremities q2h
vitamins and folic acid. c. Memory deficits c. Some hair will be removed with a razor to place
b. Thiamine helps to reverse the malnutrition often d. Paranoia electrodes. 54. The nurse is assessing the client with a tentative
associated with alcohol abuse. d. Have blood drawn for a glucose level 2 hours before diagnosis of meningitis. Which findings should the nurse
c. Thiamine reduces the risk of seizures occurring during 45. A new medication regimen is prescribed for a client the test. associate with meningitis? Select all that apply.
withdrawal from alcohol. with Parkinson’s disease. At which time should the I. Nuchal rigidity
d. Thiamine prevents neuropathy and confusion nurse make certain that the medication is taken? 50. The nurse in the ED documents that the newly II. Severe headache
associated with chronic alcohol use. a. At bedtime admitted client is “postictal upon transfer." What did III. Pill-rolling tremor
b. All at one time the nurse observe? IV. Photophobia
c. Two hours before mealtime a. Yellowing of the skin due to a liver condition V. Lethargy
a. I, II, III c. Structure 65. The nurse assesses that the client with hemolytic 71. Standards of nursing practice serve as guide for:
b. I, III, IV, and V d. Criteria anemia has weakness, fatigue, malaise, and skin and a. Nursing practice in the different fields of nursing
c. I and III only mucous membrane pallor. Which finding should the b. Proper nursing approaches and techniques
d. I, II, IV and V 60. The following are basic steps in the controlling nurse also associate with hemolytic anemia? c. Safe nursing care and management
process of the department. Which of the following is a. Scleral jaundice d. Evaluation of nursing cared rendered
55. The nurse is assessing the client diagnosed with NOT included? b. A smooth, red tongue
meningococcal meningitis. Which assessment data a. Measure actual performance c. A craving for ice to chew Situation
would warrant notifying the HCP? b. Set nursing standards and criteria d. A poor intake of fresh vegetables You are taking care of Mrs. Gil, 65 years old, who is
a. Purpuric lesions on the face c. Compare results of performance to standards and terminally ill with ovarian cancer stage IV.
b. Complaints of light hurting the eyes objectives 66. The client is being admitted with folic acid
c. Dull, aching, frontal headache d. Identify possible courses of action deficiency anemia. Which would be the most 72. When caring for a dying client you will perform
d. Not remembering the day of the week appropriate referral? which of the following activities?
61. Which of the following statements refers to criteria? a. Alcoholics anonymous a. Encourage the client to reach optimal health
56. Which type of precautions should the nurse a. Agreed on level of nursing care b. Leukemia society of the PH b. Assist client perform activities of daily living
implement for the client diagnosed with septic b. Characteristics used to measure the level of nursing c. A hematologist c. Assist the client towards a peaceful death
meningitis? care d. A social worker d. Motivate client to gain independence
a. Standard precautions c. Step-by-step guidelines
b. Airborne precautions d. Statement which guide the group in decision making Situation 73. The client prepares for eventual death and
c. Contact precautions and problem solving The nurse's understanding of ethico-legal discusses with the nurse and her family how she would
d. Droplet precautions responsibilities will guide his/her nursing practice. like her funeral to look like and what dress she will use.
Situation This client is in the stage of:
Situation Using Maslow's need theory, Airway, Breathing and 67. The principles that -govern right and proper a. Acceptance
The Mariano Marcos Memorial Hospital and Medical Circulation are the physiological needs vital to life. The conducts of a person regarding life, biology and the b. Resolution
Center just opened its new Performance Improvement nurse's knowledge and ability to identify and health professions is referred to as: c. Denial
Department. Mr. Greg is appointed as the Quality immediately intervene to meet these needs is a. Morality d. bargaining
Control Officer. He commits himself to his new role and important to save lives. b. Religion
plans his strategies to realize the goals and objectives of c. Values 74. The spouse of a client dying from lung cancer states,
the department. 62. Which of these clients has a problem with the d. Bioethics “ I don’t understand this death rattle. She has not had
transport of oxygen from the lungs to the tissues? anything to drink in days. Where is the fluid coming
57. Which of the following is a primary task that they a. Carol with a tumor in the brain 68. The purpose of having nurses' code of ethics is: from? Which is the hospice care nurse’s best response?
should perform to have an effective control system? b. Theresa with anemia a. Delineate the scope and areas of nursing practice a. “The body produces about two teaspoons of fluid
a. Make an interpretation about strengths and c. Sonny Boy with a fracture in the femur b. Identify nursing action recommended for specific every minute on its own.”
weaknesses d. Brigette with diarrhea healthcare situations b. “Are you sure someone is not putting ice chips in her
b. Identify the values of the department c. To help the public understand professional conduct, mouth?”
c. Identify structure, process, outcome standards & 63. Laboratory tests are prescribed for the client who expected of nurses c. “There is no reason for this, but it does happen from
criteria has a smooth and reddened tongue and ulcers at the time to time.”
d. Measure actual performances corners of the mouth. Which result would the nurse 69. You inform the patient about his rights which d. “I can administer a patch to her skin to dry up the
find if the client has iron-deficiency anemia? include the following EXCEPT: secretions if you wish.”
58. Ms. Valencia develops the standards to be followed. a. Low hemoglobin and hematocrit a. Right to expect reasonable continuity of care
Among the following standards, which is considered as b. Elevated red blood cells (RBCs) b. Right to consent to or decline to participate in 75. The hospice care nurse is conducting a spiritual care
a structure standard? c. Prolonged prothrombin time (PT) research studies or experiments assessment. Which statement is the scientific rationale
a. The patients verbalized satisfaction of the nursing d. Elevated white blood cells (WBCs) c. Right to obtain information about another patient for this intervention?
care received d. Right to expect that the records about his care will be a. The client will ask all of his or her spiritual questions
b. Rotation of duty will be done every four weeks for all 64. The nurse is teaching the client who is a strict treated as confidential and get answers.
patient care personnel. vegetarian how to decrease the risk of developing b. The nurse is able to explain to the client how death
c. All patients shall have their weights taken recorded megaloblastic anemia. Which information should the 70. The principle states that a person has unconditional will affect the spirit.
d. Patients shall answer the evaluation form before nurse provide? worth and has the capacity to determine his own c. Spirituality provides a sense of meaning and purpose
discharge a. Undergo an annual Schilling test. destiny. for many clients.
b. Increase intake of foods high in iron. a. Bioethics d. The nurse is the expert when assisting the client with
59. When she presents the nursing procedures to be c. Supplement the diet with vitamin B12 b. Justice spiritual matters.
followed, she refers to what type of standards? d. Have a hemoglobin level drawn monthly. c. Fidelity
a. Process d. Autonomy 76. The client who is terminally ill called the significant
b. Outcome others to the room and said good-bye, then dismissed
them and now lies quietly and refuses to eat. The nurse Which self-care measures should the nurse
understands the client is in what stage of the grieving 82. After stabilizing the client’s cervical spine, which 87. Nurse Oni should conduct a focused assessment recommend? Select all that apply.
process? action should the nurse take next? with client with multiple sclerosis for risk of which of I. “Adductor spasms can be relieved by taking a
a. Denial a. Carefully remove the driver from the car. the following? hot bath."
b. Anger b. Assess the client’s pupil for reaction. I. Dehydration II. “If a muscle is in spasm, stretch and hold it,
c. Bargaining c. Assess the client’s airway. II. Falls and then relax.”
d. Acceptance d. Attempt to wake the client up by shaking him. III. Seizures III. “Rest first and then walk as able using a
IV. Skin breakdown walker for support.“
Situation 83. In assessing the client with T12 SCI, which clinical V. Fatigue IV. “When walking, keep feet close together,
Nursing research is considered essential to the manifestations would the nurse expect to find to a. II, III, IV, V legs slightly bent.”
achievement of high-quality patient care and outcomes. support the diagnosis of spinal shock? b. I, II, III, IV V. “Set an alarm to remind you to void 30
a. No reflex activity below the waist c. II, IV, V minutes After fluid intake."
77. Which of the following is a method of non- b. Inability to move upper extremities d. I and II a. II, III, V
probability sampling? c. Complaints of a pounding headache b. II, III, IV
a. Cluster sampling d. Hypotension and bradycardia 88. Which of the following is not a typical clinical c. II, III, I
b. Snowball sampling manifestation of MS? d. II, III
c. Simple random sampling 84. The client with a C6 SCI is admitted to the a. Double vision
d. stratified random sampling emergency department complaining of a sever b. Sudden bursts of energy Situation
pounding headache and has BP of 180/110. Which c. Weakness in the extremities Nurse Carla is caring for a patient with Guillain-Barré.
78. The data on the family’s number of children is intervention should the emergency department nurse d. Muscle tremors
appropriate for what level of measurement? implement? 92. The client diagnosed with Guillain-Barré syndrome is
a. Interval a. Keep the client flat in bed 89. Nurse Oni should know that the primary reason why scheduled to receive plasmapheresis treatments. The
b. Ordinal b. Dim the lights in the room she find it difficult to evaluate the effectiveness of the client’s spouse asks the nurse about the purpose of
c. Ratio c. Assess for bladder distention drugs the client has used for 15 years is? plasmapheresis. Which explanation is correct?
d. Nominal d. Administer a narcotic analgesic. a. The client exhibits intolerance to many drugs. a. “Plasmapheresis removes excess fluid from the
b. The client experiences spontaneous remissions from bloodstream.”
79. A nurse researcher wants to study the response of 85. The nurse assesses the client, who was injured in a time to time. b. “Plasmapheresis will increase the protein levels in the
patients who suffer from dysrhythmia to pacemaker. diving accident 2 hours earlier. The client is breathing c. The client requires multiple drugs simultaneously. blood.”
The appropriate research design would be? independently but has no movement or muscle tone d. The client endures long periods of exacerbation c. “Plasmapheresis removes circulating antibodies from
a. Experimental research design from below the area of injury. A CT scan reveals a before the illness responds to a particular drug. the blood.”
b. Descriptive-correlational fracture of the C4 cervical vertebra. The nurse should d. “Plasmapheresis infuses lipoproteins to restore the
c. Comparative descriptive plan interventions for which problem? 90. The client with MS tells the nurse about extreme myelin sheath.”
d. Correlational a. Complete spinal cord transection fatigue. Which assessment findings should the nurse
b. Spinal shock identify as contributing to the client's fatigue? Select all 93. The nurse is caring for the client experiencing
c. An upper motor neuron injury that apply. Guillain-Barré syndrome (GBS). It is most important for
80. Which of the following are qualitative data sources? d. Quadriplegia I. Hemoglobin 9.5 g/dL and hematocrit is 31.8% the nurse to monitor the client for which complication?
a. Interview and observation. II. Taking baclofen 15 mg 3 times per day a. Autonomic dysreflexia
b. Primary sources and secondary sources 86. Spinal precautions are ordered for the client. Who III. Working 4 to 8 hours per week in the family b. Septic emboli
c. Books and journals sustained a neck injury during an MVA. The client has business c. Cardiac dysrhythmias
d. Questionnaires and survey yet to be cleared that there is no cervical fracture. IV. Stopped taking amitriptyline 8 weeks earlier d. Respiratory failure
Which action is the nurse’s priority when receiving the V. Presence of a cardiac murmur at the tricuspid
81. All of the following are advantages of using client in the ED? valve.
questionnaires EXCEPT: a. Assessing the client using the Glasgow Coma Scale VI. Bilateral leg weakness noted when walking 94. Nurse Carla learns that the pathophysiology of
a. Easy to test data for reliability and validity (GCS) in room Guillain-Barré syndrome includes segmental
b. Facilitates data gathering b. Assessing the level of sensation in the client‘s a. I, II, III
c. Less time consuming than interview and observation demyelination. The nurse should understand that this
extremities b. I, II, III, IV
d. Respondents may provide socially acceptable c. Checking that the cervical collar was correctly placed c. I, II, IV, V, VI causes what?
answers by EMS d. I, II, III, IV, V a. Delayed afferent nerve impulses
d. Applying antiembolism hose to the client‘s lower b. Paralysis of affected muscles
Situation Extremities 91. The home-care nurse is counseling the client who c. Paresthesia in upper extremities
Nurse Jenny arrives at the site of a one-car motor- has MS. The client is experiencing weakness, ataxia, d. Slowed nerve impulse transmission
vehicle accident and stops to render aid. The driver of Situation intermittent adductor spasms of the hips, and
the car is unconscious. Nurse Oni is caring for patients with Multiple Sclerosis. occasional incontinence from loss of bladder sensation.
95. Which assessment finding is most indicative of a. Paranoia
b. Flashbacks
Guillain-Barré syndrome (GBS)? c. Gastric disturbances
A. Pupillary dilation d. Conjunctival infection
B. Expressive aphasia
C. Loss of bowel and bladder control 100. The client states, “I don’t see any problem with
D. A sudden onset of muscle weakness and pain smoking a little weed. It isn’t addictive.”
Which response by the nurse is most accurate?
96. Which laboratory result is consistent with the a. “Marijuana is a natural chemical that has many
therapeutic uses, but it is still illegal to use.”
diagnosis of Guillain-Barré syndrome (GBS)? b. “Marijuana is not addictive. The danger is that. it
A. Positive rheumatoid factor often leads to abuse of more illicit drugs.”
B. Decreased serum albumin c. “Marijuana has effects similar to alcohol,
C. Decreased erythrocyte sedimentation rate hallucinogens, and sedatives that are addictive.”
D. Increased protein in the cerebrospinal fluid d. “There are no withdrawal symptoms, so it is
controversial whether marijuana is addictive.”
Situation
Nurse Mira works as a nurse in a rehabilitation center
for individuals who abuse a certain substance.

97.The spouse of the client who is currently in inpatient


treatment for substance abuse tells the nurse, “We’ve
done this so many times. I don’t think my spouse is ever
going to change. Do you think it’s time for me to get a
divorce?” Which response by the nurse is most helpful?
a. “You don’t think your spouse is ever going to
change?”
b. “Sounds like you’re feeling discouraged in your
marriage.”
c. “Your spouse will likely continue to use and need
treatment again.”
d. “That’s your decision; I can’t tell you whether you
should get a divorce.”

98. The nurse is in the working phase of a relationship


with the client being treated for substance abuse.
Which intervention would be appropriate during this
phase of treatment?
a. Assessing the client’s readiness to change substance-
abusing behavior
b. Evaluating the effectiveness of the client’s newly
adapted coping skills
c. Confronting the client’s denial that substances have
negatively impacted daily life
d. Determining the extent to which substances have
impaired the client’s functioning

99. The nurse is assessing the college student who


presents with generalized fatigue, dry mouth,
tachycardia, and an increased appetite. Which
additional finding from the client’s history and physical
exam should alert the nurse to explore possible
marijuana abuse?

You might also like